SlideShare ist ein Scribd-Unternehmen logo
1 von 11
PRE-BOARD
PROFESSIONAL EDUCATION (Elementary)
`
MULTIPLE CHOICE
1. All of the following describe the development of children aged
eleven to thirteen EXCEPT
A .they show abstract thinking and judgement
B. they exhibit increased objectivity in thinking
C. sex differences in IQ becomes more evident
D. they shift from impulsivity to adaptive ability
2. A teacher’s summary of a lesson serves the following functions,
EXCEPT
A. it links the parts of the lesson.
B. it clinches the basic ideas or concepts of the lesson.
C. it makes provisions for full participation of students.
D. it brings together the information that has been discussed.
3. The principle of the individual difference requires teachers to
___________.
A. gives greater attention to gifted learners
B. prepare modules for slow learners in class
C treat all learners alike while in the classroom
D. provide for a variety of learning activities
4. After giving an input on a good paragraph, Teacher W asks her
students to rate a given paragraph along the elements of a good
paragraph. The students’ task is in level of _________.
A. application B. evaluation C. synthesis D. analysis
5. John Watson said “Men are built not born. ” What does this
statement point to?
A. The ineffectiveness of training on a person’s development.
B. The absence of genetic influence on a person’s development.
C. The effect of heredity.
D. The effect of environmental stimulation on a person’s development.
6. Which is a sound classroom management practice?
A. Apply rules and policies on a case to case basis.
B. Apply reactive approach to discipline.
C. Establish routines for all daily needs and tasks.
D. Avoid establishing routines; routines make your student robots.
7. How can you exhibit legitimate power on the first day of school?
A. By making them feel you have mastery of subject matter.
B. By making them realize the importance of good grades.
C. By making your students feel they are accepted for who they are.
D. By informing them you are allowed to act in loco parents.
8. Which assumption underlines the teacher’s use of performance
objectives?
A. Performance objectives assure the learner of learning.
B. The success of the learner is based on teacher’s performance.
C. Learning is defined as a change in the learner’s observable
performance.
D. Not every form of learning id observable.
9. For maximum interaction, a teacher ought to avoid _____ questions.
A. leading B. divergent C. rhetorical D. informational
10. In a social studies class, Teacher I present a morally ambiguous
situation and asks his students what they would do. On whose theory
is Teacher I’s technique based?
A. Piaget B. Kohlberg C. Bruner D. Bandura
11. A child who gets punished for stealing candy may not steal again
immediately. But this does not mean that the child may not steal
again. Based on Thorndike’s theory on punishment and learning, this
shows that ______________.
A. punishment weakens a response
B. punishment does not remove a response
C. punishment remove a response
D. punishment strengthen e a response
12. Based on Piaget’s theory, what should a teacher provide for
children in the concrete operational stage?
A. Activities for hypothesis formulation.
B. Games and other physical activities to develop motor skills.
C. Learning activities that involve problems of classification and
ordering.
D. Stimulating environment with ample objectives to play with.
13. All subjects in Philippine elementary and secondary schools are
expected to be taught using the integrated approach. This came about
as a result of implementation of ___________.
A. School-Based Management
B. Program for Decentralized Education
C. Basic Education Curriculum
D. Schools First Initiative
14. A goal-oriented instruction culminates in _______.
A. planning activities B. formulation of objectives
C. identification of topics D. evaluation
15. Student Z does not study at all but when the Licensure
Examination for Teachers comes, before he takes the LET, he spends
one hour or more praying for a miracle, i.e, to pass the examination.
Which attitude towards religion or God is displayed?
A. Religion as authentic C. Religion as magic
B. Religion as real D. Religion as fake
16. As a teacher, what do you do when you engage yourself in major
task analysis?
A. Test if learning reached higher level thinking skills
B. Breakdown a complex task into sub-skills
C. Determine the level of thinking involved
D. Revise lesson objectives
17. During the Spanish period, what was/were the medium/media of
instruction in schools?
A. English C. Spanish
B. Spanish and the Vernacular D. The Vernacular
18. In instructional planning it is necessary that parts of the plan from
the first to the last have ___________.
A. conciseness B. coherence C. clarity D. symmetry
19. Behavior followed by pleasant consequences will be strengthened
and will be more likely to occur in the future. Behavior followed by
unpleasant consequences will be weakened and will be more likely to
be repeated in the future. Which one is explained?
A. Freud’s psychoanalytic theory
B. Bandura’s social learning theory
C. Thorndike’s law effect
D. B.F. Skinner’s Operant conditioning theory
20. What was the prominent educational issues of the mid 1980’s?
A. Mainstreaming C. Inductive Reasoning
B. Value Education D. Modeling
21. Which teaching activity is founded on Bandura’s social learning
theory?
A. Questioning C. Bilingual Education
B. Lecturing D. Accountability
22. A mother gives his boy his favorite snack everytime the boy
cleans up his room. Afterwards, the boy cleaned his room everyday
in anticipation of the snack. Which theory is illustrated?
A. Classical conditioning C. Operant conditioning
B. Associative learning D. Pavlonian conditioning
23. The concepts of Trust vs. mistrust, autonomy vs. shame & self-
doubt, and initiative vs. guilt are most closely related with works of
_____.
A. Jung B. Freud C. Erickson D. Piaget
24. Ruben is very attached to his mother and Ruth to her father. In
what developmental stage are they according to Freudian
psychological theory?
A. Latent stage C. Anal Stage
B. Pre-genital stage D. Oedipal stage
25. What does extreme authoritarianism in the home reinforce in
learners?
A. Creativity in work. B. Dependence on others for directions
C. Ability to direct themselves D. Doing things on their own initiative
26. Which types of play is most characteristic of a four to six-year old
child?
A. Cooperative and solidarity plays
B. Solidarity plays and onlookers plays
C. Associative and onlookers plays
D. Associative and cooperative plays
27. The main purpose of compulsory study of the Constitution is
to______________.
A. makes constitutional experts of the students
B. develop students into responsible, thinking citizens
C. acquaint student with the historical development of the Phil
Constitution
D. prepare students for law-making
28. Which does Naom Chomsky assert about language learning for
children?
I. Young children learn and apply grammatical rules and vocabulary as
they are exposed to them.
II. Begin formal teaching of grammatical rules to children as early as
possible.
III. Do not require initial formal language teaching for children.
A. I and II B. II only C. I only D. I and III
29. With which goals of educational institutions as provided for by the
Constitution is the development of work skills aligned?
A. To develop moral character
B. To Teach the duties of citizenship
C. To develop vocational efficiency
D. To inculcate love of country
30. Soc exhibits fear response to freely roaming dogs but does not
show fear when a dog is on a leash or confined to a pen. Which
conditioning process is illustrated?
A. Acquisition C. Discrimination
B. Generation D. Extinction
31. Teacher F is convinced that whatever a student performs a
desired behavior, provided reinforcement and soon the student will
learn to perform the behavior on his own. On which principle is
Teacher F’s conviction based?
A. Constructivism C. Environmentalism
B. Behaviorism D. Cognitivism
32. Studies in the areas of neurosciences discloses that the human
brain has limitless capacity. What does this imply?
A. Every pupil has his own native ability and his learning is limited to
this native ability.
B. Every child is a potential genius.
C. Pupil can possibly reach a point where they have learned
everything.
D. Some pupils are admittedly not capable of learning.
33. Based on Piaget’s theory, what should a teacher provide for
children in the sensimotor stage?
A. Games and other physical activities to develop motor skill.
B. Activities for hypothesis formulation.
C. Learning activities that involve problems of classification and
ordering.
D. Stimulating environment with ample objects to play with.
34. Bruner’s theory on intellectual development moves from enactive
to iconic and symbolic stages. In which stage(s) are diagrams helpful
to accompany verbal information?
A. Symbolic C. Symbolic and enactive
B. Enactive and iconic D. Iconic
35. Researches conducted show that teacher’s expectations of
students become self-fulfilling prophecies. What is this phenomenon
called?
A. Halo effect C. Hawthorne effect
B. Pygmalion effect D. Ripple effect
36. Rodel is very aloof and cold in his relationship with classmates.
Which basic goals must have not been attained by Rodel during his
developmental years, according to Erickson’s theory of psychological
development?
A. Autonomy B. Initiative C. Generativity D. Trust
37. Which one may support equitable access but may sacrifice
quality?
A. Selective retention C. School accreditation
B. Deregulated tuition fee hike D. Open admission
38. Based on Freud’s psychoanalytic theory which component (s)
personality is (are) concerned with a sense of right and wrong?
A. Super ego and Ego C. Id
B. Super-ego D. Ego
39. It is not wise to laugh at a two-year old child when he utters bad
word because in his stage he is learning to __________.
A. socialize C. distinguish right from wrong
B. considered other’s views D. distinguish sex differences
40. In a treatment for alcoholism, Ramil was made to drink an
alcoholic beverage and then made to ingest a drug that produces
nausea. Eventually, h nauseated at he sight and smell of alcohol and
stopped drinking alcohol. Which theory explains this?
A. Operant conditioning C. Associative learning
B. Social learning theory D. Attribution theory
41. Theft of school equipment like t.v. computer, etc. by teenagers in
the community itself is becoming a common phenomenon. What
does this incident signify?
2
A. Prevalence of poverty in the community.
B. Inability of school to hire security guards.
C. Community’s lack of sense of co-ownership.
D. Deprivation of Filipino schools.
42. Teacher B engages her students with information for thorough
understanding, for meaning and for competent application. Which
principle governs Teacher B’s practice?
A. Behaviorist B. Gestalt C. Cognitivist D. Constructivist
43. Under which program were students who were not accommodated
in public elementary and secondary schools because of lack of
classroom, teachers, and instructional materials, were enrolled in
private schools in their respective communities at the government’s
expense?
A. Government Assistance Program
B. National Scholarship Program
C. Educational Service Contract System
D. Study Now-Pay later
44. Which is/are the sources of man’s intellectual drives, according to
Freud?
A. Super ego B. Id C. Id –ego D. Ego
45. A student passes a research report poorly written but ornately
presented in a folder to make up for the poor quality of the book report
content. Which Filipino trait does this practice prove?
A. art of academics C. “porma” over substance
B. art over science D. substance over “porma”
46. Which is NOT a sound purpose for asking questions?
A. To encourage self-reflection
B. To probe deeper after an answer is given
C. To remind students of a procedure
D. To discipline a bully in class
47. “A stitch on time saves nine”, so goes the adage. Applied to
classroom management, this means that we _______.
A. must be reactive in our approach to discipline
B. may not occupy ourselves with disruptions which are worth ignoring
because they are minor
C. have to resolve minor disruptions before they are out of control
D. may apply 9 rules out of 10 consistently
48. In Krathwoh’s taxonomy of objectives in the affective, which is
most authentic?
A. Responding C. Valuing
B. Organization D. Characterization
49. Teacher G’s lessons objective has something to do with the skill of
synthesizing? Which behavioral term is most appropriate?
A. Test B. Appraisal C. Assess D. Theorize
50. Teacher H strives to draw participation of every student into
classroom discussion. Which student’s need is she trying to address?
The need
A. to get everything out in the open
B. to feel significant and be part of a group
C. to be creative
D. to show their oral abilities to the rest of the class
51. A sixth grade twelve-year old boy comes from a dysfunctional
family and has been abused and neglected. He has been to two
orphanages and three different elementary schools. The student can
decode on the second grade level, but he can comprehend orally
material at the fourth or fifth grade level. The most probable cause/s of
this student’s reading problem is/are ______.
A. poor teaching C. neurological factors
B. emotional factors D. immaturity
52. Teacher B clears his throat to communicate disapproval of a
student’s behavior. Which specific influence technique is this?
A. Proximity control C. Signal interference
B. Direct appeal D. Interest boosting
53. Which questioning practice promotes more class interaction?
A. Focusing on convergent questions.
B. Focusing on divergent questions.
C. Asking rhetorical questions
D. Asking the question before calling a student
54. The following are used in writing performance objectives EXCEPT
A. integrate B. diagram C. delineate D. comprehend
55. Research tells that teachers ask mostly content questions. Which
of the following terms does NOT refer to content question?
A. Direct B. Convergent C. Concept D. Closed
56. Which is behavioral term describes a lesson outcome in the
highest level of Bloom’s cognitive domain?
A. Design B. Create C. Evaluate D. Analyze
57. Which is an appropriate way to make manage off-task behavior?
A. Make eye contact.
B. Stop your class activity to correct a child who is no longer on task.
C. Move closer to the child.
D. Redirect a child’s attention to task and check his progress to make
sure he is continuing to work.
58. How can you exhibit referent power on the first day of school?
A. By reminding your students your authority over them again and
again.
B. By giving your students a sense of belonging and acceptance.
C. By making them feel you know what you are talking about.
D. By telling them the importance of good grades.
59. If a teacher plans a constructivist lesson, what will he most likely
do? Plan how he can _______.
A. do evaluate his student’s work
B. do reciprocal teaching
C. do lecture to his students
D. engage his students in convergent thinking
60. How can you exhibit expert power on the first day of school?
A. By making them realize the importance of good grades.
B. By giving your students a sense of belonging and acceptance.
C. By reminding then your students your authority over them again and
again.
D. By making them feel you know what you are talking about.
61. Read this question: “How will you present the layers of the earth to
your class?” This is a question that _________.
A. assesses cognition C. probes creative thinking
B. directs D. leads the students to evaluate
62. In Krathwoh’s affective domain of objectives, which of the following
is the lowest level of effective behavior?
A. Characterization C. Organization
B. Responding D. Valuing
3
63. The following are sound specific purpose of questions EXCEPT:
A. To stimulate learners to ask questions
B. To arouse interest and curiosity
C. To call the attention of an inattentive student
D. To teach via student answers
64. An effective classroom manager uses low-profile classroom
control. What is a low-profile classroom technique?
A. Note to parents C. Withdrawal of privileges
B. Raising the pitch of the voice D. After-school detention
65. Test norms are established in order to have a basis for ______.
A. identifying pupil’s difficulties C. establishing learning goals
B. computing grades D. interpreting test results
66. Quiz to formative test while periodic is to _________.
A. criterion-reference test C. norm-reference test
B. summative test D. diagnostic test
67. Two students are given the WISC III. One has a full scale IQ of 91,
while the other has an IQ of 109. Which conclusion can be drawn?
A. The first student is probably below average, while the second has
above average potential.
B. Both students are functioning in the average range of intellectual
ability
C. Another IQ test should be given to truly assess their intellectual
potential.
D. The second student has significantly higher intellectual ability.
68. Student’s scores on a test were: 72,72,73,74,76,78,81,83,85. The
score 76 is the ___.
A. mode B. mean C. median D. average
69. The best way for a guidance counselor to begin to develop study
skills and habits in underachieving student would be to ______.
A. have them view filmstrips about various study approaches
B. encourage students to talk study habits from their own experiences
C. give out list of effective study approaches
D. have these underachieving students observe the study habits of
excelling students.
70. Which describes norm-referenced grading?
A. What constitutes a perfect score C. An absolute standard
B. The performance of the group D. The student’s past performance
71. Standard deviation is to variability as mode to _____.
A. correction C. central tendency
B. discrimination D. level of difficulty
72. If teacher wants to test student’s ability to organize ideas, which
type of test should she formulate?
A. Short answer C. Essay
B. Technical problem type D. Multiple-choice type
73. What is the mean of this score distribution 4,5,6,7,8,9,10?
A. 6 B. 7 C. 8.5 D. 7.5
74. Study this group of tests which was administered with the
following results, then answer the question.
Subject Mean SD Ronnel’s scrore
Math 56 10 43
Physics 41 9 31
English 80 16 109
In which subject(s) did Ronnel perform best in relation to the group’s
performance?
A. Physics B. English C. Math D. Physics and Math
75. Out of 3 distracters in a multiple choice test items, namely B,C,
and D, no pupil choice D as answer. This implies that D is ________.
A. a plausible distracter B. a vague distracter
C. an effective distracter D. an ineffective distracte
76. Shown a picture of children in sweaters inside the classrooms, the
students were asked this question: In what kind of climate do these
children live? This is a thought questions on ______.
A. predicting B. applying C. creating D. inferring
77.Which one can best evaluate student’s attitudinal development?
A. Short answer test C. Essay test
B. Observation D. Portfolio
78. In which competency do my students find the greater difficulty? In
item with a difficulty index of
A. 0.9 B. 0.1 C. 1.0 D. 0.5
79. With synthesizing skills in mind, which has the highest diagnostic
value?
A. Performance test C. Completion test
B. Multiple choice test D. Essay test
80. “In the light of the facts presented, what is most likely to happen
when…?” is a simple thought question on _____.
A. synthesizing B. generalizing C. inferring D. justifying
81. Which is most implied by negatively skewed score distribution?
A. The score are evemly distributed from left to the right.
B. Most pupils are achievers.
C. Most of the scores are low.
D. Most of the scores are high.
82. Study this group of tests which was administered with the
following results, then answer the question.
Subject Mean SD Ronnel’s scrore
Math 56 10 43
Physics 41 9 31
English 80 16 109
In which subject(s) were the scores most homogenous?
A. English C. Physics and Math
B. Math D. Physics
83. Which guidance in test construction is NOT observed in this test
item “Jose Rizal wrote ____”.
A. The alternative must be plausible.
B. There must be only one correct answer.
C. The central problem should be packed in the stem.
D. Alternatives must grammatical parallelism.
84. Are percentile ranks the same as percentage correct?
A. Yes
B. It cannot be determined unless scores are given.
C. It cannot be determined unless the number of examinees is given
D. No
85. “What is most likely to happen to our economy when export
continuously surpasses import” is a thought question on ______.
A. creating C. synthesizing
B. relating cause-and-effect D. predicting
4
86. The burnout malady gets worse if a teacher doesn’t intervene to
change whatever areas he or she in control. Which one can renew a
teacher’s enthusiasm?
A. Judge someone else as wrong. C. Stick to job.B. Initiate
changes in jobs. D. Engage in self-pity.
87. In the context on the theory on multiple intelligences, what is one
weakness of the paper-pencil test?
A. It lacks reliability.
B. It is not easy to administer.
C. It utilizes so much time.
D. It puts the non-linguistically intelligent at a disadvantage
88. Which holds true to standardized tests?
A. They are scored according to different standards.
B. They are administered differently.
C. They are used for assigned grades.
D. They are used for comparative purposes.
89. The first thing to do in constructing a periodic test is for a teacher to
A. decide on the type of test to construct
B. study the content
C. decide on the number of items for the test
D. go back to her instructional objectives
90. Which can effectively measure student’s awareness of values?
A. Moral dilemma C. Anecdotal record
B. Projective techniques D. Likert scales
91. I drew learners into several content areas and encouraged them
to solve a complex question for interdisciplinary teaching. Which
strategy did I use?
A. Thematic instruction C. Unit method
B. Problem-centered learning D. Reading-writing activity
92. With specific details in mind, which one has (have) a stronger
diagnostic value?
A. Restricted essay test
B. Non-restricted essay test
C. Restricted and non-restricted essay tests
D. Multiple choice test
93. If your Licensure Examination for Teacher (LET) items sample
adequately the competencies listed in the syllabi, it can be said that
LET possesses ______ validity.
A. concurrent B. content C. predictive D. construct
94. In a criterion-referenced testing, what must you do to ensure that
your test is fair? .
A. Ask each student to contribute one question.
B. Use objectives for the unit as guide in your test construction.
C. Make twenty questions but ask the students to answer only ten of
their choice.
D. Make all of the questions true or false.
95. In his second item analysis, Teacher H found out that more from
the lower group got the test item # 6 correctly. This means that the
test item _______
A. has a lower validity.
B. has a positive discriminating power.
C. has a high reability
D. has a negative discriminating power
96. Study this group of tests which was administered with the
following results, then answer the question.
Subject Mean SD Ronnel’s scrore
Math 56 10 43
Physics 41 9 31
English 80 16 109
In which subject(s) did Ronnel perform poorly in relation to the group’s
performance?
A. Physics C. English
B. Math D. English and Math
97. To promote effective practice, which guidelines should you bear in
mind? Practice should be ______.
A. done in an evaluative atmosphere
B. difficult for students to learn a lesson
C. arranged to allow student to receive feedback
D. take place over a long period of time
98. Teacher A discovered that his pupils are very good in dramatizing.
Which tool must have helped him discover his pupils’ strength?
A. Portfolio assessment C. Performance test
B. Paper-and-pencil test D.Journal entry
99. By what name is Socratic method also known?
A. Mastery learning C. Questioning Method
B. Morrison method D. Indirect instruction
100. Teacher F wanted to teach the pupils the skills to do cross
stitching. He check up quiz was a written test on the steps of cross
stitching. What characteristic of a good test does it lack?
A. Reliability B. Scorability C. Objectivity D. Validity
101. Which method has been proven to be effective in courses that
stress acquisition of knowledge?
A. Cooperative learning C. Mastery learning
B. Socratic method D. Indirect instruction
102. Teacher Y does norm-referenced interpretation of scores. Which
of the following does she do?
A. She describes group performance in relation to a level of mastery
set.
B. She uses a specified content as its frame of reference.
C. She compares every individual student’s scores with others’ scores.
D. She describes what should be their performance.
103 . Which is a major advantage of a curriculum-based assessment?
A. It is based on an norm-referenced measurement model.
B. It tends to focus on anecdotal information on student progress.
C. It connects testing with teaching.
D. It is informal in nature.
104. Which is the first step in planning an achievement test?
A. Select the type of test items to use.
B. Decide on the length of the test
C. Build a table of specification
D. Define the instructional objective
105. Which test has broad sampling of topics as strength?
A. Short answer test C. Problem test
B. Objective test D. Essay test
106. Which is one characteristic of an effective classroom
management?
A. It respects cultural norms of a limited group students.
B. It quickly and unobtrusively redirects misbehavior once it occurs.
C. Strategies are simple enough to be used consistently.
D. It teaches dependence on other for self-control.
107. Which guideline must be observed in the use of prompting to
shape the correct performance of your students?
A. Refrain form using prompts.
B. Use the most intrusive prompt first.
C. Use the least intrusive prompt first.
5
D. Use all prompts available.
108. To elicit more student’s response, Teacher G. made use of
covert responses. Which one did she NOT do?
A. She refrained from judging on the student’s responses.
B. She had the students write their responses privately then called
each of them.
C. She showed the correct answers on the overhead after the
students have written their responses.
D. She had the students write their response privately.
109. In self-directed learning, to what extent should a teacher’s
“scaffolding” be?
A. None, to force the student to learn by himself
B. To the maximum, in order to extend to the student all the help he
needs.
C. To the minimum, to spend up development of student’s sense of
independence.
D. To a degree the student needs it.
110. Read the following then answer the questions:
TEACHER: IN WHAT WAYS OTHER THAN THE PERIODIC
TABLE MIGHT WE PREDICT THE
UNDISCOVERED ELEMENTS?
BOBBY: WE COULD GO TO THE MOON AND SEE
IF THERE ARE SOME ELEMENTS THERE
WE DON’T HAVE.
BETTY: WE COULD DIG DOWN TO THE CENTER
OF THE EARTH AND SEE IF WE FIND ANY
OF THE MISSING ELEMENTS.
RICKY: WE COULD STUDY DEBRIS FROM THE
METEORITES- IF WE CAN FIND ANY.
TEACHER: THOSE ARE ALL GOOD ANSWERS BUT
WHAT IF THOSE EXCURSIONS TO THE
MOON, TO THE CENTER OF THE EARTH
OR TO FIND METEORITES WERE TOO
COSTLY AND TIME CONSUMING? HOW
MIGHT WE USE THE ELEMENTS WE
ALREADY HAVE HERE ON EARTH TO
FIND SOME NEW ONES?
Question: which questioning strategy/ies does/do the exchange
of thoughts above illustrate?
A. Extending and lifting B. Funneling
C. Sowing and reaping D. Nose-dive
111. Which are direct measures of competence?
A. Standardized tests B. personality tests
C. Performance tests D. paper-and-pencil tests
112. Availment of the Philippine Education Placement Test for adults
and out-of-school youths is in support of the government’s educational
program towards ___ .
A. relevance B. quality and relevance
C. equitable access D. quality
113. Teacher T taught a lesson denoting ownership by means of
possessives. He first introduced the rule, then gave examples,
followed by class exercises, then back to the rule before he moved
into second rule. Which presenting technique did he use?
A. Combinatorial B. Sequential
C. Comparative D.. Part-whole
114. Which Filipino trait works against the shift in teacher’s role from
teacher as a fountain of information to teacher as facilitator?
A. Pakikisama B. Authoritarianism
C. Authoritativeness D. Hiya
115. Referring to Teacher S, Nicolle describes her teacher as “fair,
caring and someone you can talk to”. Which power or leadership does
Teacher S have?
A. Expert power B. Reward power
C. Referent power D. Legitimate power
116. Teacher B uses the direct instruction strategy. Which sequence
of steps will she follow?
I. Independent practice II. Feedback and correctiveness
III. Guided student practice IV. Presenting and structuring
V. Reviewing the previous day’s work
A. III-II-IV-I-V B. I-V-II-III-IV
C. V-II-IV-III-I D. V-IV-III-II-I
117. The teacher’s first task in the selection of media in teaching is to
determine the
A. technique to be used B. availability of the media
C. objectives of the lesson D. choice of the students
118. Teacher M’s pupils are quite weak academically and his lesson
is already far behind time table. How should Teacher M proceed with
his lesson?
A. Experientially B. Deductively C. Inductively D. Logically
119In mastery learning, the definition of an acceptable standard of
performance is called a
A. behavior B. SMART
C. condition D. criterion measure
120. What values are being given priority by juries in criminal cases?
A. The rights of the criminal over the strict interpretation of the law
B. The safety of the community over sympathy for the criminal
C. The punishment of the criminal over the safety of the community
D. The needs of the criminals over the advice of the judge
121. Helping in the development of graduates who are “maka-Dios” is
an influence of ___.
A. dialectical morality C. situational morality
B. naturalistic morality D. classical Christian morality
122. What should you do if a parent who is concerned about a grade
his child received compared to another student’s grade, demands to
see both students’ grades?
A. Refuse to show any record without expressing permission from
principal.
B. Show only his child’s record.
C. Show both records to him. D. Refuse to show either record.
123. A teacher / student is held responsible for his actions because
s/he _______.
A. has reason B. has instincts C. has a choice D. is mature
124. With indirect instruction in mind, which does NOT belong to the
group?
A. Problem solving C. Lecture-recitation
B. Discovery D. Inductive reasoning
125. A teacher who equates authority with power does NOT usually
__________.
A. retaliate C. shame
B develop self-respect in every pupil D. intimidate
126. The typical autocratic teacher consistently does the following
EXCEPT
A. ridiculing students C. encouraging students
B. intimidating students D. shaming students
127. Teacher H and teacher I are rivals for promotion. To again the
6
favor of the promotion staff, teacher I offers her beach resort for free
for members of he promotional staff before ranking. As one of the
contenders for promotions, is this becoming of her to do?
A. Yes. The rare invitation will certainly be welcomed by an
overworked promotional staff
B. Yes. There’s nothing wrong with sharing one’s blessings.
C. Yes. This will be professional growth for the promotional staff
D. No. This may exert undue influence on the members of the
promotional staff and so may fail to promote on the basis of merit.
128. Teacher A knows of the illegal activities of a neighbor but keeps
quiet in order not to be involved in any investigation. Which
foundational principle of morality does Teacher A fail to apply?
A. Between two evils, do the lesser evil
B. The principle of double effect
C. The end does not justify the means
D. Always do what is right
129. Which is a true foundation of the social order?
A. Equitable distribution of wealth
B. Strong political leadership
C. The reciprocation of rights and duties
D. Obedient citizenry
130. Teacher F is a newly converted to a religion. Deeply convinced of
his new found religion, he starts Monday classes by attacking one
religion and convinces his pupil to attend their religion services on
Sundays. Is this in accordance with the Code of Ethics of Professional
Teachers?
A. Yes. In the name of academic freedom, a teacher can decide what
to teach.
B. Yes. What he does strengthens value education.
C. No. A teacher should not use his position to proselyte others.
D. Yes. What he does is a value education.
131. Teacher Q does not want Teacher B to be promoted and so
writes an anonymous letter against Teacher B accusing her of
fabricated lies. Teacher Q mails this anonymous letter to School
Division Superintendent. What should Teacher Q do if she has to act
professionally?
A. Hire a group to distribute poison letters against Teacher B for the
information dissemination.
B. Submit a signed justifiable criticism against Teacher B, if there is
any.
C. Go straight to the School Division Superintendent and gives
criticism verbally.
D. Instigate student activists to read poison letter over the microphone
132. Rights and duties are correlative. This means that.
A. rights and duties arise from natural law
B. rights and duties regulate the relationship of men in society
C. rights and duties ultimately come from God
D, each right carries with it one or several corresponding duties
133. In the Preamble of Code of Ethics of Professional Teachers,
which is NOT said of teachers?
A. Possess dignity and reputation
B. With high moral values as well as technical and professional
competence
C. Duly licensed professionals
D. LET passer
134. In what way can teachers uphold the highest possible standard of
quality education?
A. By working out undeserved promotions
B. By wearing expensive clothes to change people’s poor perception of
teachers
C. By putting down other professions to lift the status of teaching
D. By continually improving themselves personally and professionally
135. If you agree with Rizal on how you can contribute to our nation’s
redemption, which should you work for?
A. Gaining economic recovery
B. Stabilizing the political situation
C. Opening our doors to foreign influence
D. Upgrading the quality of the Filipino through education
136. In a study conducted, the pupils were asked which nationality they
preferred. If given a choice. Majority of the pupils wanted to
Americans. In this case, in which obligation, relative to the state, do
schools seem to be failing? In their obligation to
A. respect for all duly constituted authorities
B. install allegiance to the Constitution
C. promote obedience to the laws of the state
D. promote national pride
137. History books used in schools are replete with events portraying
defeats and weaknesses of the Filipino as a people. How should you
tackle them in the classroom?
A. Present them and express your feelings of shame.
B. Present facts and use them as means in inspiring your class to learn
from them.
C. Present them and blame those people responsible or those who
have contributed.
D. Present them as they are present and tell the class to accept reality.
138. Teacher often complain of numerous non-teaching assignments
that adversely affect their teaching. Does this mean that teachers
must be preoccupied only with teaching?
A. Yes, because other community leaders, not leaders, not teachers,
are asked to lead in community activities.
B. Yes, because teaching is enough full time job.
C. Yes, if they are given other assignments, justice demands that they
be properly compensated
D. No, because every teacher is expected to provide leadership and
initiative in activities for betterment of communities.
139. Each teacher is said to be a trustee of the cultural and
educational heritage of the nation and is under obligation to transmit to
learners such heritage. Which practice makes him fulfill such
obligation?
A. Observing continuing professional education.
B. Use the latest instruction technology.
C. Study the life of Filipino heroes.
D. Use interactive teaching strategies
140. Teacher A is directed to pass an undeserving student with a
death threat. Which advice will a hedonist give?
A. Pass the student. That will be use to the student, his parents and
you.
B. Don’t pass him. You surely will not like someone to give you a
death threat in order to pass.
C. Pass the student. Why suffer the threat?
D. Don’t pass him. Live by principle of justice. You will get reward, if
not in this life, in the next.
141. Teacher P wants to develop the skill of synthesizing in her pupil.
Which one will she do?
A. Directs her students to ask questions on the parts of the lesson not
understood.
B. Tell her pupils to state data presented in graphs.
C. Ask her students to formulate a generalization from the data shown
in graphs.
D. Ask her students to answer questions beginning with “What if …”
142. Which measure(s) of central tendency separate(s) the top half of
the group from the bottom half?
A. Median and mean B. Mode C. Mean D. Media
143. Which applies when skewness is zero?
7
A. Median is greater than mean.
B. Mean is greater than the median.
C. Scores have three modes.
D. Scores are normally distributed.
144. Based on Edgar Dale’s Cone of Experience, which activity is
farthest from the real thing?
A. Attend exhibit B. View images C. Hear D. Read
145. NSAT and NEAT results are interpreted against set mastery level.
This means that NSAT and NEAT fall under _______.
A. aptitude test C. criterion-referenced test
B. norm-referenced test D. intelligence test
146. Which one can enhance the comparability of grades?
A. Individual teachers giving weights to factors considered for rating.
B. Formulating tests that vary from one teacher to another.
C. Allowing individual teachers to determine factors for rating
D. Using common conversation table for translating test scores in to
ratings.
147. Read the following then answer the questions:
TEACHER: IN WHAT WAYS OTHER THAN THE PERIODIC
TABLE MIGHT WE PREDICT THE UNDISCOVERED
ELEMENTS?
BOBBY: WE COULD GO TO THE MOON AND SEE
IF THERE ARE SOME ELEMENTS THERE WE DON’T HAVE.
BETTY: WE COULD DIG DOWN TO THE CENTER
OF THE EARTH AND SEE IF WE FIND ANY OF THE MISSING
ELEMENTS.
RICKY: WE COULD STUDY DEBRIS FROM THE
METEORITES-IF WE CAN FIND ANY.
TEACHER: THOSE ARE ALL GOOD ANSWERS. BUT WHAT
IF THOSE EXCURSIONS TO THE MOON, TO THE CENTER
OF THE EARTH OR TO FIND METEORITES WERE TOO
COSTLY AND TIME CONSUMING? HOW MIGHT WE USE
THE ELEMENTS WE ALREADY HAVE HERE ON EARTH TO
FIND SOME NEW ONES?
Question: The Teacher questions in the above exchange are
examples of ____ questions.
A.Direct B. Concept C. Closed D. Fact
148. I combined several subject areas in order to focus on a
single concept for interdisciplinary teaching. Which
strategy/method did I use?
A. Reading–writing activity C. Thematic instruction
B. Unit method D. Problem-entered learning
149. “All men are pretty much alike. It is only by custom that they are
set apart, “ said one Oriental philosopher. Where can this thought be
most inspiring?
A. In a class composed of indigenous people.
B. In heterogeneous class of learners.
C. In a multi-cultural group of learners.
D. In a multi-cultural and heterogeneous groups of learners and
indigenous peoples’ group.
150. I want to teach concepts, patterns and abstractions. Which
method is most appropriate?
A. Discovery C. Direct instruction
B. Problem solving D. Indirect instruction
151. What should a teacher do for students in his class who are on
grade level?
A. Mohammed B. Confucius C. Buddha D. Lao tsu
152. Who among the following needs less verbal counseling but needs
more concrete operational forms of assistance? The child who ______.
A. has attention-deficit disorder C. has mental retardation
B. has learning disability D. has conduct disorder
153. Teacher B is a teacher of English as Second Language: she uses
vocabulary cards, fill-in-the blank sentences, dictation and writing
exercises in teaching a lesson about grocery shopping. Based on this
information, which of the following is a valid conclusion?
A. The teacher is emphasizing listening and speaking skills.
B. The teacher wants to do less talk.
C. The teacher is reinforcing learning by giving the same information in
a variety of methods.
D. The teacher is applying Bloom’s hierarchy of cognitive learning.
154. Which illustrates a development approach in guidance and
counseling?
A. Acting as a mentor
B. Making the decision for the confused student
C. Spotting on students in need of guidance
D. Teaching students how to interact in a positive manner
155. Which technique should a teacher use to encourage response if
his students do not respond to his question?
A. Ask a specific student to respond, state the question, and wait a
response.
B. Tell the class that it will have detention unless answers are
forthcoming.
C. Wait for a response.
D. Ask another question, an easier one.
156. In the parlance of test construction what does TOS mean?
A. Table of Specifics C. Table of Specification
B. Term of Specifications D. Table of Specific Test Items
157. Based on Edgar Dale’s Cone of Experience, which activity is
closest from the real thing?
A. View images B. Hear C. Watch a demo D. Attend exhibit
158. For which may you use the direct instruction method?
A. Become aware of the pollutants around us.
B. Use a microscope properly.
C. Distinguish war from aggression.
D. Appreciate Milton’s Paradise Lost.
159. Why should a teacher NOT use direct instruction all the time?
A. It requires much time.
B. It is generally effective only in the teaching of concepts and
abstractions.
C. It requires use of many supplemental materials.
D. It reduces student’s engagement in learning.
160. What measure/s of central tendency do/es the number 16
represent in the following data: 14,15,17,16,19,20,16,14,16?
A. Median B. Mode and mean C. Mode D. Median
161. Teacher E discussed how electricity flows through wires and what
generates the electric charge. Then she gave the students wires,
bulbs, switches, and dry cells and told the class to create a circuit that
will increase the brightness of each bulb. Which one best describes
the approach used?
A. It helped students understand scientific methodology.
B. It was contructivist.
C. It used a taxonomy of basic thinking skills.
D. It used cooperative learning.
162. Teacher B is a teacher of English as Second Language: she uses
vocabulary cards, fill-in-the blank sentences, dictation and writing
8
exercises in teaching a lesson about grocery shopping. Based on this
information, which of the following is a valid conclusion?
A. The teacher wants to make her teaching easier by having less talk.
B. The teacher is emphasizing reading and writing skills.
C. The teacher is applying Bloom’s hierarchy of cognitive learning
D. The teacher is teaching in a variety of ways because not all
students learn in the same manner.
163. Standard deviation is to variability as mean is to _______.
A. level of difficulty C. coefficient of correlation
B. central tendency D. discrimination index
164. Which is a form of direct instruction?
A. Problem solving C. Programmed instruction
B. Discovery process D. Inductive reasoning
165. Value clarification as a strategy in Values Education classes is
anchored on which philosophy?
A. Hedonism C. Idealism
B. Existentialism D. Christian philosophy
166. Which type of reports to “on-the spot” description of some
ncident, episode or occurrence that is being observed and recorded as
being of possible significance?
A. Value and interest report C. Biographical report
B. Anecdotal report D. Autobiographical report
167. Direct instruction id a facts, rules, and actions as indirect
instruction is for _____ ,____, _____.
A. hypotheses, verified data and conclusions
B. concepts, processes and generalizations
C. concepts, patterns and abstractions
D. guesses, data and conclusions
168. Which does NOT belong to the group of alternative learning
systems?
A. Multi-grade grouping C. Multi-age grouping
B. Non-graded grouping D. Graded education
169. Which activity should a teacher have more for his students if he
wants to develop logical-mathematical thinking?
A. Choral reading C. Problem solving
B. Storytelling D. Drama
170. Which behavior is exhibited by a student who is strong in
interpersonal intelligence?
A. Works on his/her own.
B. Keeps interest to himself/herself
C. Seeks out a classmate for help when problem occurs.
D. Spends time meditating.
171. If teacher has to ask more higher-order questions, he has to ask
more ____ questions.
A. closed B. Fact C. convergent D. concept
172. Which criterion should guide a teacher in the choice of
instructional devices?
A. Novelty B. Appropriateness C. Cost D. Attractiveness
173. The primary objective of my lesson is: “ To add similar fractions
correctly.” This is the advice of the ______.
A. Primary B. Major C. Terminal D. Enabling
174. With-it-ness, according to Kourin, is one of the characteristics of
an effective classroom manager. Which phrase goes with it?
A. Have hands that write fast.
B. Have a mouth ready to spea
C. Have eyes on the back of your hands
D. Have minds packed with knowledge.
175. Which is one role play in the pre-school and early childhood
years?
A. Separates reality from fantasy.
B. Develops competitive spirit.
C. Develops the upper and lower limbs
D. Increase imagination due to expanding knowledge and emotional
range.
176. A guest in one graduation rites told his audience: “Reminder, you
are what you choose to be”. The guest speaker is more of a/an _____.
A. idealism B. existentialist C. realistic D. pragmatist
177. Principal C shares this thought with the teachers: Subject matter
should help students understand and appreciate themselves as unique
individual who accept complete responsibility for their thoughts,
feelings and actions. From which philosophy is this thought based?
A. Essentialism B. Perennialism C. Progressivism D. Existentialism
178. Which of the following prepositions is attributed to Plato?
A. Sense perception is the most accurate guide to knowledge.
B. Learning is the discovery of truth as latent ideas are brought to
consciousness
C. Truth is relative to a particular time and place.
D. Human beings create their own truths.
179. Student B claims: “I cannot see perfection but I long for it. So it
must be real. “under which group can he be classified?
A. Realist B. Empiricist C. Pragmatist D. Idealist
180. Teacher U teaches to his pupils that pleasure is not the highest
good. Teacher’s teaching is against what philosophy?
A. Realism B. Hedonism C. Empiricism D. Epicureanism
181. Who among the following puts more emphasis on core
requirements, longer school day, longer academic year and more
challenging textbooks?
A. Progressivist B. Perennialist C. Existentialist D. Essentialist
182. To come closer to the truth we need to “go back to the things
themselves.” This is the advice of the ______.
A. pragmatists B. idealist C. phenomenologists D. behaviorist
183. Teacher W wants to review and check on the lesson of the
previous day. Which one will be most reliable?
A. Sampling the understanding of a few students
B. Explicitly reviewing the task-relevant information necessary for the
day’s lesson.
C. Having students correct each other’s work.
D. Having students identify difficult homework problems.
184. Whose influence is the education program that puts emphasis on
self-development through the classics, music, and ritual?
A. Mohammed B. Confucius C. Buddha D. Lao tsu
185. Teacher H gave first-grade class a page with a story in which
picture take the place of some words. Which method did she use?
A. The Spaulding method B. The language experience approach
C. The rebus method D. The whole language approach
9
186. The search for related literature by accessing several data bases
by the use of a telephone line to connect a computer that have
database is termed ______.
A. computer search C. manual search
B. on-line search D. compact disc search
187. On whose philosophy was A. S. Neil Summerhill, one of the most
experimental schools based?
A. Pestalozzi B. John Locke C. Montessori D. Rousseau
188. You arrive at knowledge by re-thinking of latent ideas. From
whom does this thought come?
A.Existentialism B. Experimentalist C. Idealist D. Realist
189. As a teacher, you are rationalist, Which among these will be your
guiding principle?
A. I must teach the child that we can never have real knowledge of
anything.
B. I must teach the child to develop his mental powers to the full.
C. I must teach the child so he is assured of heaven
D. I must teach the child every knowledge, skills and value that he
needs for a better future.
190. Principal B tells her teachers that training in the humanities is
most important. To which educational philosophy does he adhere?
A. Progressivism B. Existentialism C. Essentialism D. Perennialism
191. As a teacher, you are reconstructionist, Which among these will be your
guiding principle?
A. I must teach the child so he is assured of heaven
B. I must teach the child every knowledge, skills and value that he needs for a
better future.
C. I must teach the child to develop his mental powers to the full.
D. I must teach the child that we can never have real knowledge of anything.
192. What can be said of Peter who obtained a score of P75 in a
Grammar objective test?
A. His rating is 75.
B. He answered 75 items in the test correctly.
C. He answered 75% of the test items correctly.
D. He performed better that 5% of his classmate.
193. Which group of philosophers maintain the “truth exists in an
objective order that is independent of the knower”?
A. Idealists B. Pragmatists C. Existentialists D. Realists
194. If we teach our students to think creativity, what do we encourage
them to do?
A. To question the illogical B. To criticize the unreasonable
C. Think “within the box” D. Do “outside-the-box” thinking
195. Which of the following is considered a peripheral device?
A. Keyboard B. CPU C. Monitor D. Printer
196. Helping in the development of graduates who are “maka-Diyos” is
an influence of ___.
A. naturalistic morality B. situational morality
C. dialectical morality D. classical Christian morality
197. In research which is another term for independent variable?
A. Response B. Outcome C. Criterion D. Input
198. Which practices speak of education of the human spirit?
I. Familiarizing the learner with the world’s heritage of art in all its
forms
II. Encouraging the learner to be practical
III. Studying the biographies of heroes and martyr
IV. Teaching the learner how to learn
A. I, II B. I,IV C. II,III D. I, III
199. Rights cannot be unduly tramped upon or suppressed without
moral guit because they are
A. Inalienable. C. perfect.
B. Inviolable. D. adventitious.
200. Which instructional material is closest to direct experience?
A. Film showing C. Field trip
B. Simulation D. Dramatization
10
11

Weitere ähnliche Inhalte

Was ist angesagt?

DLL-september 19-23 adverb - .docx
DLL-september 19-23 adverb - .docxDLL-september 19-23 adverb - .docx
DLL-september 19-23 adverb - .docxEdnaAlaba1
 
Narrative in Practice Teaching
Narrative in Practice TeachingNarrative in Practice Teaching
Narrative in Practice TeachingJenica20
 
Fs 1 (learning episode 4)
 Fs 1 (learning episode 4) Fs 1 (learning episode 4)
Fs 1 (learning episode 4)HaikoMitshi
 
Elements of a short story.Lesson Plan
Elements of a short story.Lesson PlanElements of a short story.Lesson Plan
Elements of a short story.Lesson PlanYen Bunsoy
 
My Practice Teaching - E narrative presentation
My Practice Teaching - E narrative presentationMy Practice Teaching - E narrative presentation
My Practice Teaching - E narrative presentationAileen Anastacio
 
COT Lesson Plan in English 3 Verbs
COT Lesson Plan in English 3 VerbsCOT Lesson Plan in English 3 Verbs
COT Lesson Plan in English 3 Verbswinzfred
 
Criteria in Choosing Appropriate Assessment Tool
Criteria in Choosing Appropriate Assessment ToolCriteria in Choosing Appropriate Assessment Tool
Criteria in Choosing Appropriate Assessment ToolIra Sagu
 
Teaching And Learning In The 21st Century
Teaching And Learning In The 21st CenturyTeaching And Learning In The 21st Century
Teaching And Learning In The 21st Centuryguest224aa9
 
Licensure examination for teachers
Licensure examination for teachersLicensure examination for teachers
Licensure examination for teachersMina Olario
 
K to 12 Grade 3 MATHEMATICS NAT (National Achievement Test)
K to 12 Grade 3 MATHEMATICS NAT (National Achievement Test) K to 12 Grade 3 MATHEMATICS NAT (National Achievement Test)
K to 12 Grade 3 MATHEMATICS NAT (National Achievement Test) LiGhT ArOhL
 
Technology Integration Matrix Introduction
Technology Integration Matrix IntroductionTechnology Integration Matrix Introduction
Technology Integration Matrix IntroductionAmy G.
 
Field Study 2 Episode 6
Field Study 2 Episode 6Field Study 2 Episode 6
Field Study 2 Episode 6Jundel Deliman
 
A narrative report on teaching experiences
A narrative report on teaching experiencesA narrative report on teaching experiences
A narrative report on teaching experiencesYuna Lesca
 

Was ist angesagt? (20)

DLL-september 19-23 adverb - .docx
DLL-september 19-23 adverb - .docxDLL-september 19-23 adverb - .docx
DLL-september 19-23 adverb - .docx
 
Narrative in Practice Teaching
Narrative in Practice TeachingNarrative in Practice Teaching
Narrative in Practice Teaching
 
Field study 1 episode 2
Field study 1 episode 2Field study 1 episode 2
Field study 1 episode 2
 
Fs 1 (learning episode 4)
 Fs 1 (learning episode 4) Fs 1 (learning episode 4)
Fs 1 (learning episode 4)
 
Elements of a short story.Lesson Plan
Elements of a short story.Lesson PlanElements of a short story.Lesson Plan
Elements of a short story.Lesson Plan
 
My Practice Teaching - E narrative presentation
My Practice Teaching - E narrative presentationMy Practice Teaching - E narrative presentation
My Practice Teaching - E narrative presentation
 
COT Lesson Plan in English 3 Verbs
COT Lesson Plan in English 3 VerbsCOT Lesson Plan in English 3 Verbs
COT Lesson Plan in English 3 Verbs
 
Portfolio
PortfolioPortfolio
Portfolio
 
Criteria in Choosing Appropriate Assessment Tool
Criteria in Choosing Appropriate Assessment ToolCriteria in Choosing Appropriate Assessment Tool
Criteria in Choosing Appropriate Assessment Tool
 
Teaching And Learning In The 21st Century
Teaching And Learning In The 21st CenturyTeaching And Learning In The 21st Century
Teaching And Learning In The 21st Century
 
Field study 2
Field study 2Field study 2
Field study 2
 
Field study portfolio
Field study portfolioField study portfolio
Field study portfolio
 
Licensure examination for teachers
Licensure examination for teachersLicensure examination for teachers
Licensure examination for teachers
 
K to 12 Grade 3 MATHEMATICS NAT (National Achievement Test)
K to 12 Grade 3 MATHEMATICS NAT (National Achievement Test) K to 12 Grade 3 MATHEMATICS NAT (National Achievement Test)
K to 12 Grade 3 MATHEMATICS NAT (National Achievement Test)
 
Fs 5 Learning Assessment Strategies
Fs 5 Learning Assessment StrategiesFs 5 Learning Assessment Strategies
Fs 5 Learning Assessment Strategies
 
Gened science
Gened scienceGened science
Gened science
 
Technology Integration Matrix Introduction
Technology Integration Matrix IntroductionTechnology Integration Matrix Introduction
Technology Integration Matrix Introduction
 
FS1-EP07-EP08-2021.pdf
FS1-EP07-EP08-2021.pdfFS1-EP07-EP08-2021.pdf
FS1-EP07-EP08-2021.pdf
 
Field Study 2 Episode 6
Field Study 2 Episode 6Field Study 2 Episode 6
Field Study 2 Episode 6
 
A narrative report on teaching experiences
A narrative report on teaching experiencesA narrative report on teaching experiences
A narrative report on teaching experiences
 

Ähnlich wie Sample preboard-exam-prof ed

Let professional education 3
Let professional education 3Let professional education 3
Let professional education 3Alex Acayen
 
Professional education set d (without highlighted answers)
Professional education set d (without highlighted answers)Professional education set d (without highlighted answers)
Professional education set d (without highlighted answers)Lucille Clavero
 
Professional education set b (with highlighted answers)
Professional education set b (with highlighted answers)Professional education set b (with highlighted answers)
Professional education set b (with highlighted answers)Lucille Clavero
 
Let professional education 8
Let professional education 8Let professional education 8
Let professional education 8Alex Acayen
 
Licensure examination for teachers set 1 part 1
Licensure examination for teachers set 1 part 1Licensure examination for teachers set 1 part 1
Licensure examination for teachers set 1 part 1Arneyo
 
Licensure examination for teachers set 1 part 1
Licensure examination for teachers set 1 part 1Licensure examination for teachers set 1 part 1
Licensure examination for teachers set 1 part 1Arneyo
 
Reviewer Let professional education 13
Reviewer Let professional education 13Reviewer Let professional education 13
Reviewer Let professional education 13Daniel Bragais
 
Licensure examination for teachers
Licensure examination for teachersLicensure examination for teachers
Licensure examination for teachersMina Olario
 
Licensure examination for teachers set 2 part 1
Licensure examination for teachers set 2 part 1Licensure examination for teachers set 2 part 1
Licensure examination for teachers set 2 part 1Arneyo
 
LET Sample Exam Professional education set a
LET Sample Exam Professional education set aLET Sample Exam Professional education set a
LET Sample Exam Professional education set abmsg wap
 
Professional education set a
Professional education set aProfessional education set a
Professional education set achinnex23
 
Professional education set a (with highlighted answers)
Professional education set a (with highlighted answers)Professional education set a (with highlighted answers)
Professional education set a (with highlighted answers)Lucille Clavero
 
Let professional education 2
Let professional education 2Let professional education 2
Let professional education 2Alex Acayen
 
Final coaching questions in prof. education by prof. cris paner
Final coaching questions in prof. education by prof. cris panerFinal coaching questions in prof. education by prof. cris paner
Final coaching questions in prof. education by prof. cris panerArneyo
 
Final coaching questions in prof. education by prof. cris paner
Final coaching questions in prof. education by prof. cris panerFinal coaching questions in prof. education by prof. cris paner
Final coaching questions in prof. education by prof. cris panerArneyo
 

Ähnlich wie Sample preboard-exam-prof ed (20)

Let. sumay
Let. sumayLet. sumay
Let. sumay
 
Let professional education 3
Let professional education 3Let professional education 3
Let professional education 3
 
Professional education set d (without highlighted answers)
Professional education set d (without highlighted answers)Professional education set d (without highlighted answers)
Professional education set d (without highlighted answers)
 
LET Reviewer
LET ReviewerLET Reviewer
LET Reviewer
 
Professionaleducationset a
 Professionaleducationset a Professionaleducationset a
Professionaleducationset a
 
Professional education set b (with highlighted answers)
Professional education set b (with highlighted answers)Professional education set b (with highlighted answers)
Professional education set b (with highlighted answers)
 
Let professional education 8
Let professional education 8Let professional education 8
Let professional education 8
 
Licensure examination for teachers set 1 part 1
Licensure examination for teachers set 1 part 1Licensure examination for teachers set 1 part 1
Licensure examination for teachers set 1 part 1
 
Licensure examination for teachers set 1 part 1
Licensure examination for teachers set 1 part 1Licensure examination for teachers set 1 part 1
Licensure examination for teachers set 1 part 1
 
Prof ed-a
Prof ed-aProf ed-a
Prof ed-a
 
Reviewer Let professional education 13
Reviewer Let professional education 13Reviewer Let professional education 13
Reviewer Let professional education 13
 
Licensure examination for teachers
Licensure examination for teachersLicensure examination for teachers
Licensure examination for teachers
 
Licensure examination for teachers set 2 part 1
Licensure examination for teachers set 2 part 1Licensure examination for teachers set 2 part 1
Licensure examination for teachers set 2 part 1
 
LET Sample Exam Professional education set a
LET Sample Exam Professional education set aLET Sample Exam Professional education set a
LET Sample Exam Professional education set a
 
Professional education set a
Professional education set aProfessional education set a
Professional education set a
 
ProfEd Drills
ProfEd DrillsProfEd Drills
ProfEd Drills
 
Professional education set a (with highlighted answers)
Professional education set a (with highlighted answers)Professional education set a (with highlighted answers)
Professional education set a (with highlighted answers)
 
Let professional education 2
Let professional education 2Let professional education 2
Let professional education 2
 
Final coaching questions in prof. education by prof. cris paner
Final coaching questions in prof. education by prof. cris panerFinal coaching questions in prof. education by prof. cris paner
Final coaching questions in prof. education by prof. cris paner
 
Final coaching questions in prof. education by prof. cris paner
Final coaching questions in prof. education by prof. cris panerFinal coaching questions in prof. education by prof. cris paner
Final coaching questions in prof. education by prof. cris paner
 

Mehr von John Paul Intano

Mehr von John Paul Intano (20)

Ict -computer_hardware_servicing_curriculum_guides_for_grades_7_to_10
Ict  -computer_hardware_servicing_curriculum_guides_for_grades_7_to_10Ict  -computer_hardware_servicing_curriculum_guides_for_grades_7_to_10
Ict -computer_hardware_servicing_curriculum_guides_for_grades_7_to_10
 
Ict k12 paul
Ict k12 paulIct k12 paul
Ict k12 paul
 
Finalictcomputerhardwareservicinggrades7 10-140422213202-phpapp02
Finalictcomputerhardwareservicinggrades7 10-140422213202-phpapp02Finalictcomputerhardwareservicinggrades7 10-140422213202-phpapp02
Finalictcomputerhardwareservicinggrades7 10-140422213202-phpapp02
 
Automotive 8 exam
Automotive 8 examAutomotive 8 exam
Automotive 8 exam
 
Word copy-for-reviewer-let
Word copy-for-reviewer-letWord copy-for-reviewer-let
Word copy-for-reviewer-let
 
Visual arts
Visual artsVisual arts
Visual arts
 
Visual arts
Visual artsVisual arts
Visual arts
 
Tle review-drafting-carpentry-masonry-plumbing-electricity
Tle review-drafting-carpentry-masonry-plumbing-electricityTle review-drafting-carpentry-masonry-plumbing-electricity
Tle review-drafting-carpentry-masonry-plumbing-electricity
 
Tle exam-drill
Tle exam-drillTle exam-drill
Tle exam-drill
 
Review profed-part-b-with-answers-1
Review profed-part-b-with-answers-1Review profed-part-b-with-answers-1
Review profed-part-b-with-answers-1
 
Ncbts LET REVIEWER
Ncbts LET REVIEWERNcbts LET REVIEWER
Ncbts LET REVIEWER
 
Majorship tle-electronics
Majorship tle-electronicsMajorship tle-electronics
Majorship tle-electronics
 
Majorship tle-3
Majorship tle-3Majorship tle-3
Majorship tle-3
 
Majorship tle-2
Majorship tle-2Majorship tle-2
Majorship tle-2
 
Majorship tle-1
Majorship tle-1Majorship tle-1
Majorship tle-1
 
Let2013 march-1
Let2013 march-1Let2013 march-1
Let2013 march-1
 
Let reviewer
Let reviewerLet reviewer
Let reviewer
 
Hele
HeleHele
Hele
 
Electricity
ElectricityElectricity
Electricity
 
General education-natural-science
General education-natural-scienceGeneral education-natural-science
General education-natural-science
 

Kürzlich hochgeladen

Food safety_Challenges food safety laboratories_.pdf
Food safety_Challenges food safety laboratories_.pdfFood safety_Challenges food safety laboratories_.pdf
Food safety_Challenges food safety laboratories_.pdfSherif Taha
 
Python Notes for mca i year students osmania university.docx
Python Notes for mca i year students osmania university.docxPython Notes for mca i year students osmania university.docx
Python Notes for mca i year students osmania university.docxRamakrishna Reddy Bijjam
 
Basic Civil Engineering first year Notes- Chapter 4 Building.pptx
Basic Civil Engineering first year Notes- Chapter 4 Building.pptxBasic Civil Engineering first year Notes- Chapter 4 Building.pptx
Basic Civil Engineering first year Notes- Chapter 4 Building.pptxDenish Jangid
 
Making communications land - Are they received and understood as intended? we...
Making communications land - Are they received and understood as intended? we...Making communications land - Are they received and understood as intended? we...
Making communications land - Are they received and understood as intended? we...Association for Project Management
 
ICT Role in 21st Century Education & its Challenges.pptx
ICT Role in 21st Century Education & its Challenges.pptxICT Role in 21st Century Education & its Challenges.pptx
ICT Role in 21st Century Education & its Challenges.pptxAreebaZafar22
 
2024-NATIONAL-LEARNING-CAMP-AND-OTHER.pptx
2024-NATIONAL-LEARNING-CAMP-AND-OTHER.pptx2024-NATIONAL-LEARNING-CAMP-AND-OTHER.pptx
2024-NATIONAL-LEARNING-CAMP-AND-OTHER.pptxMaritesTamaniVerdade
 
Understanding Accommodations and Modifications
Understanding  Accommodations and ModificationsUnderstanding  Accommodations and Modifications
Understanding Accommodations and ModificationsMJDuyan
 
Single or Multiple melodic lines structure
Single or Multiple melodic lines structureSingle or Multiple melodic lines structure
Single or Multiple melodic lines structuredhanjurrannsibayan2
 
General Principles of Intellectual Property: Concepts of Intellectual Proper...
General Principles of Intellectual Property: Concepts of Intellectual  Proper...General Principles of Intellectual Property: Concepts of Intellectual  Proper...
General Principles of Intellectual Property: Concepts of Intellectual Proper...Poonam Aher Patil
 
This PowerPoint helps students to consider the concept of infinity.
This PowerPoint helps students to consider the concept of infinity.This PowerPoint helps students to consider the concept of infinity.
This PowerPoint helps students to consider the concept of infinity.christianmathematics
 
Unit-V; Pricing (Pharma Marketing Management).pptx
Unit-V; Pricing (Pharma Marketing Management).pptxUnit-V; Pricing (Pharma Marketing Management).pptx
Unit-V; Pricing (Pharma Marketing Management).pptxVishalSingh1417
 
Accessible Digital Futures project (20/03/2024)
Accessible Digital Futures project (20/03/2024)Accessible Digital Futures project (20/03/2024)
Accessible Digital Futures project (20/03/2024)Jisc
 
SKILL OF INTRODUCING THE LESSON MICRO SKILLS.pptx
SKILL OF INTRODUCING THE LESSON MICRO SKILLS.pptxSKILL OF INTRODUCING THE LESSON MICRO SKILLS.pptx
SKILL OF INTRODUCING THE LESSON MICRO SKILLS.pptxAmanpreet Kaur
 
Spellings Wk 3 English CAPS CARES Please Practise
Spellings Wk 3 English CAPS CARES Please PractiseSpellings Wk 3 English CAPS CARES Please Practise
Spellings Wk 3 English CAPS CARES Please PractiseAnaAcapella
 
Wellbeing inclusion and digital dystopias.pptx
Wellbeing inclusion and digital dystopias.pptxWellbeing inclusion and digital dystopias.pptx
Wellbeing inclusion and digital dystopias.pptxJisc
 
HMCS Max Bernays Pre-Deployment Brief (May 2024).pptx
HMCS Max Bernays Pre-Deployment Brief (May 2024).pptxHMCS Max Bernays Pre-Deployment Brief (May 2024).pptx
HMCS Max Bernays Pre-Deployment Brief (May 2024).pptxEsquimalt MFRC
 
Jamworks pilot and AI at Jisc (20/03/2024)
Jamworks pilot and AI at Jisc (20/03/2024)Jamworks pilot and AI at Jisc (20/03/2024)
Jamworks pilot and AI at Jisc (20/03/2024)Jisc
 
ICT role in 21st century education and it's challenges.
ICT role in 21st century education and it's challenges.ICT role in 21st century education and it's challenges.
ICT role in 21st century education and it's challenges.MaryamAhmad92
 
80 ĐỀ THI THỬ TUYỂN SINH TIẾNG ANH VÀO 10 SỞ GD – ĐT THÀNH PHỐ HỒ CHÍ MINH NĂ...
80 ĐỀ THI THỬ TUYỂN SINH TIẾNG ANH VÀO 10 SỞ GD – ĐT THÀNH PHỐ HỒ CHÍ MINH NĂ...80 ĐỀ THI THỬ TUYỂN SINH TIẾNG ANH VÀO 10 SỞ GD – ĐT THÀNH PHỐ HỒ CHÍ MINH NĂ...
80 ĐỀ THI THỬ TUYỂN SINH TIẾNG ANH VÀO 10 SỞ GD – ĐT THÀNH PHỐ HỒ CHÍ MINH NĂ...Nguyen Thanh Tu Collection
 

Kürzlich hochgeladen (20)

Spatium Project Simulation student brief
Spatium Project Simulation student briefSpatium Project Simulation student brief
Spatium Project Simulation student brief
 
Food safety_Challenges food safety laboratories_.pdf
Food safety_Challenges food safety laboratories_.pdfFood safety_Challenges food safety laboratories_.pdf
Food safety_Challenges food safety laboratories_.pdf
 
Python Notes for mca i year students osmania university.docx
Python Notes for mca i year students osmania university.docxPython Notes for mca i year students osmania university.docx
Python Notes for mca i year students osmania university.docx
 
Basic Civil Engineering first year Notes- Chapter 4 Building.pptx
Basic Civil Engineering first year Notes- Chapter 4 Building.pptxBasic Civil Engineering first year Notes- Chapter 4 Building.pptx
Basic Civil Engineering first year Notes- Chapter 4 Building.pptx
 
Making communications land - Are they received and understood as intended? we...
Making communications land - Are they received and understood as intended? we...Making communications land - Are they received and understood as intended? we...
Making communications land - Are they received and understood as intended? we...
 
ICT Role in 21st Century Education & its Challenges.pptx
ICT Role in 21st Century Education & its Challenges.pptxICT Role in 21st Century Education & its Challenges.pptx
ICT Role in 21st Century Education & its Challenges.pptx
 
2024-NATIONAL-LEARNING-CAMP-AND-OTHER.pptx
2024-NATIONAL-LEARNING-CAMP-AND-OTHER.pptx2024-NATIONAL-LEARNING-CAMP-AND-OTHER.pptx
2024-NATIONAL-LEARNING-CAMP-AND-OTHER.pptx
 
Understanding Accommodations and Modifications
Understanding  Accommodations and ModificationsUnderstanding  Accommodations and Modifications
Understanding Accommodations and Modifications
 
Single or Multiple melodic lines structure
Single or Multiple melodic lines structureSingle or Multiple melodic lines structure
Single or Multiple melodic lines structure
 
General Principles of Intellectual Property: Concepts of Intellectual Proper...
General Principles of Intellectual Property: Concepts of Intellectual  Proper...General Principles of Intellectual Property: Concepts of Intellectual  Proper...
General Principles of Intellectual Property: Concepts of Intellectual Proper...
 
This PowerPoint helps students to consider the concept of infinity.
This PowerPoint helps students to consider the concept of infinity.This PowerPoint helps students to consider the concept of infinity.
This PowerPoint helps students to consider the concept of infinity.
 
Unit-V; Pricing (Pharma Marketing Management).pptx
Unit-V; Pricing (Pharma Marketing Management).pptxUnit-V; Pricing (Pharma Marketing Management).pptx
Unit-V; Pricing (Pharma Marketing Management).pptx
 
Accessible Digital Futures project (20/03/2024)
Accessible Digital Futures project (20/03/2024)Accessible Digital Futures project (20/03/2024)
Accessible Digital Futures project (20/03/2024)
 
SKILL OF INTRODUCING THE LESSON MICRO SKILLS.pptx
SKILL OF INTRODUCING THE LESSON MICRO SKILLS.pptxSKILL OF INTRODUCING THE LESSON MICRO SKILLS.pptx
SKILL OF INTRODUCING THE LESSON MICRO SKILLS.pptx
 
Spellings Wk 3 English CAPS CARES Please Practise
Spellings Wk 3 English CAPS CARES Please PractiseSpellings Wk 3 English CAPS CARES Please Practise
Spellings Wk 3 English CAPS CARES Please Practise
 
Wellbeing inclusion and digital dystopias.pptx
Wellbeing inclusion and digital dystopias.pptxWellbeing inclusion and digital dystopias.pptx
Wellbeing inclusion and digital dystopias.pptx
 
HMCS Max Bernays Pre-Deployment Brief (May 2024).pptx
HMCS Max Bernays Pre-Deployment Brief (May 2024).pptxHMCS Max Bernays Pre-Deployment Brief (May 2024).pptx
HMCS Max Bernays Pre-Deployment Brief (May 2024).pptx
 
Jamworks pilot and AI at Jisc (20/03/2024)
Jamworks pilot and AI at Jisc (20/03/2024)Jamworks pilot and AI at Jisc (20/03/2024)
Jamworks pilot and AI at Jisc (20/03/2024)
 
ICT role in 21st century education and it's challenges.
ICT role in 21st century education and it's challenges.ICT role in 21st century education and it's challenges.
ICT role in 21st century education and it's challenges.
 
80 ĐỀ THI THỬ TUYỂN SINH TIẾNG ANH VÀO 10 SỞ GD – ĐT THÀNH PHỐ HỒ CHÍ MINH NĂ...
80 ĐỀ THI THỬ TUYỂN SINH TIẾNG ANH VÀO 10 SỞ GD – ĐT THÀNH PHỐ HỒ CHÍ MINH NĂ...80 ĐỀ THI THỬ TUYỂN SINH TIẾNG ANH VÀO 10 SỞ GD – ĐT THÀNH PHỐ HỒ CHÍ MINH NĂ...
80 ĐỀ THI THỬ TUYỂN SINH TIẾNG ANH VÀO 10 SỞ GD – ĐT THÀNH PHỐ HỒ CHÍ MINH NĂ...
 

Sample preboard-exam-prof ed

  • 1. PRE-BOARD PROFESSIONAL EDUCATION (Elementary) ` MULTIPLE CHOICE 1. All of the following describe the development of children aged eleven to thirteen EXCEPT A .they show abstract thinking and judgement B. they exhibit increased objectivity in thinking C. sex differences in IQ becomes more evident D. they shift from impulsivity to adaptive ability 2. A teacher’s summary of a lesson serves the following functions, EXCEPT A. it links the parts of the lesson. B. it clinches the basic ideas or concepts of the lesson. C. it makes provisions for full participation of students. D. it brings together the information that has been discussed. 3. The principle of the individual difference requires teachers to ___________. A. gives greater attention to gifted learners B. prepare modules for slow learners in class C treat all learners alike while in the classroom D. provide for a variety of learning activities 4. After giving an input on a good paragraph, Teacher W asks her students to rate a given paragraph along the elements of a good paragraph. The students’ task is in level of _________. A. application B. evaluation C. synthesis D. analysis 5. John Watson said “Men are built not born. ” What does this statement point to? A. The ineffectiveness of training on a person’s development. B. The absence of genetic influence on a person’s development. C. The effect of heredity. D. The effect of environmental stimulation on a person’s development. 6. Which is a sound classroom management practice? A. Apply rules and policies on a case to case basis. B. Apply reactive approach to discipline. C. Establish routines for all daily needs and tasks. D. Avoid establishing routines; routines make your student robots. 7. How can you exhibit legitimate power on the first day of school? A. By making them feel you have mastery of subject matter. B. By making them realize the importance of good grades. C. By making your students feel they are accepted for who they are. D. By informing them you are allowed to act in loco parents. 8. Which assumption underlines the teacher’s use of performance objectives? A. Performance objectives assure the learner of learning. B. The success of the learner is based on teacher’s performance. C. Learning is defined as a change in the learner’s observable performance. D. Not every form of learning id observable. 9. For maximum interaction, a teacher ought to avoid _____ questions. A. leading B. divergent C. rhetorical D. informational 10. In a social studies class, Teacher I present a morally ambiguous situation and asks his students what they would do. On whose theory is Teacher I’s technique based? A. Piaget B. Kohlberg C. Bruner D. Bandura 11. A child who gets punished for stealing candy may not steal again immediately. But this does not mean that the child may not steal again. Based on Thorndike’s theory on punishment and learning, this shows that ______________. A. punishment weakens a response B. punishment does not remove a response C. punishment remove a response D. punishment strengthen e a response 12. Based on Piaget’s theory, what should a teacher provide for children in the concrete operational stage? A. Activities for hypothesis formulation. B. Games and other physical activities to develop motor skills. C. Learning activities that involve problems of classification and ordering. D. Stimulating environment with ample objectives to play with. 13. All subjects in Philippine elementary and secondary schools are expected to be taught using the integrated approach. This came about as a result of implementation of ___________. A. School-Based Management B. Program for Decentralized Education C. Basic Education Curriculum D. Schools First Initiative 14. A goal-oriented instruction culminates in _______. A. planning activities B. formulation of objectives C. identification of topics D. evaluation 15. Student Z does not study at all but when the Licensure Examination for Teachers comes, before he takes the LET, he spends one hour or more praying for a miracle, i.e, to pass the examination. Which attitude towards religion or God is displayed? A. Religion as authentic C. Religion as magic B. Religion as real D. Religion as fake 16. As a teacher, what do you do when you engage yourself in major task analysis? A. Test if learning reached higher level thinking skills B. Breakdown a complex task into sub-skills C. Determine the level of thinking involved D. Revise lesson objectives 17. During the Spanish period, what was/were the medium/media of instruction in schools? A. English C. Spanish B. Spanish and the Vernacular D. The Vernacular 18. In instructional planning it is necessary that parts of the plan from the first to the last have ___________. A. conciseness B. coherence C. clarity D. symmetry 19. Behavior followed by pleasant consequences will be strengthened and will be more likely to occur in the future. Behavior followed by unpleasant consequences will be weakened and will be more likely to be repeated in the future. Which one is explained? A. Freud’s psychoanalytic theory B. Bandura’s social learning theory C. Thorndike’s law effect D. B.F. Skinner’s Operant conditioning theory 20. What was the prominent educational issues of the mid 1980’s? A. Mainstreaming C. Inductive Reasoning B. Value Education D. Modeling
  • 2. 21. Which teaching activity is founded on Bandura’s social learning theory? A. Questioning C. Bilingual Education B. Lecturing D. Accountability 22. A mother gives his boy his favorite snack everytime the boy cleans up his room. Afterwards, the boy cleaned his room everyday in anticipation of the snack. Which theory is illustrated? A. Classical conditioning C. Operant conditioning B. Associative learning D. Pavlonian conditioning 23. The concepts of Trust vs. mistrust, autonomy vs. shame & self- doubt, and initiative vs. guilt are most closely related with works of _____. A. Jung B. Freud C. Erickson D. Piaget 24. Ruben is very attached to his mother and Ruth to her father. In what developmental stage are they according to Freudian psychological theory? A. Latent stage C. Anal Stage B. Pre-genital stage D. Oedipal stage 25. What does extreme authoritarianism in the home reinforce in learners? A. Creativity in work. B. Dependence on others for directions C. Ability to direct themselves D. Doing things on their own initiative 26. Which types of play is most characteristic of a four to six-year old child? A. Cooperative and solidarity plays B. Solidarity plays and onlookers plays C. Associative and onlookers plays D. Associative and cooperative plays 27. The main purpose of compulsory study of the Constitution is to______________. A. makes constitutional experts of the students B. develop students into responsible, thinking citizens C. acquaint student with the historical development of the Phil Constitution D. prepare students for law-making 28. Which does Naom Chomsky assert about language learning for children? I. Young children learn and apply grammatical rules and vocabulary as they are exposed to them. II. Begin formal teaching of grammatical rules to children as early as possible. III. Do not require initial formal language teaching for children. A. I and II B. II only C. I only D. I and III 29. With which goals of educational institutions as provided for by the Constitution is the development of work skills aligned? A. To develop moral character B. To Teach the duties of citizenship C. To develop vocational efficiency D. To inculcate love of country 30. Soc exhibits fear response to freely roaming dogs but does not show fear when a dog is on a leash or confined to a pen. Which conditioning process is illustrated? A. Acquisition C. Discrimination B. Generation D. Extinction 31. Teacher F is convinced that whatever a student performs a desired behavior, provided reinforcement and soon the student will learn to perform the behavior on his own. On which principle is Teacher F’s conviction based? A. Constructivism C. Environmentalism B. Behaviorism D. Cognitivism 32. Studies in the areas of neurosciences discloses that the human brain has limitless capacity. What does this imply? A. Every pupil has his own native ability and his learning is limited to this native ability. B. Every child is a potential genius. C. Pupil can possibly reach a point where they have learned everything. D. Some pupils are admittedly not capable of learning. 33. Based on Piaget’s theory, what should a teacher provide for children in the sensimotor stage? A. Games and other physical activities to develop motor skill. B. Activities for hypothesis formulation. C. Learning activities that involve problems of classification and ordering. D. Stimulating environment with ample objects to play with. 34. Bruner’s theory on intellectual development moves from enactive to iconic and symbolic stages. In which stage(s) are diagrams helpful to accompany verbal information? A. Symbolic C. Symbolic and enactive B. Enactive and iconic D. Iconic 35. Researches conducted show that teacher’s expectations of students become self-fulfilling prophecies. What is this phenomenon called? A. Halo effect C. Hawthorne effect B. Pygmalion effect D. Ripple effect 36. Rodel is very aloof and cold in his relationship with classmates. Which basic goals must have not been attained by Rodel during his developmental years, according to Erickson’s theory of psychological development? A. Autonomy B. Initiative C. Generativity D. Trust 37. Which one may support equitable access but may sacrifice quality? A. Selective retention C. School accreditation B. Deregulated tuition fee hike D. Open admission 38. Based on Freud’s psychoanalytic theory which component (s) personality is (are) concerned with a sense of right and wrong? A. Super ego and Ego C. Id B. Super-ego D. Ego 39. It is not wise to laugh at a two-year old child when he utters bad word because in his stage he is learning to __________. A. socialize C. distinguish right from wrong B. considered other’s views D. distinguish sex differences 40. In a treatment for alcoholism, Ramil was made to drink an alcoholic beverage and then made to ingest a drug that produces nausea. Eventually, h nauseated at he sight and smell of alcohol and stopped drinking alcohol. Which theory explains this? A. Operant conditioning C. Associative learning B. Social learning theory D. Attribution theory 41. Theft of school equipment like t.v. computer, etc. by teenagers in the community itself is becoming a common phenomenon. What does this incident signify? 2
  • 3. A. Prevalence of poverty in the community. B. Inability of school to hire security guards. C. Community’s lack of sense of co-ownership. D. Deprivation of Filipino schools. 42. Teacher B engages her students with information for thorough understanding, for meaning and for competent application. Which principle governs Teacher B’s practice? A. Behaviorist B. Gestalt C. Cognitivist D. Constructivist 43. Under which program were students who were not accommodated in public elementary and secondary schools because of lack of classroom, teachers, and instructional materials, were enrolled in private schools in their respective communities at the government’s expense? A. Government Assistance Program B. National Scholarship Program C. Educational Service Contract System D. Study Now-Pay later 44. Which is/are the sources of man’s intellectual drives, according to Freud? A. Super ego B. Id C. Id –ego D. Ego 45. A student passes a research report poorly written but ornately presented in a folder to make up for the poor quality of the book report content. Which Filipino trait does this practice prove? A. art of academics C. “porma” over substance B. art over science D. substance over “porma” 46. Which is NOT a sound purpose for asking questions? A. To encourage self-reflection B. To probe deeper after an answer is given C. To remind students of a procedure D. To discipline a bully in class 47. “A stitch on time saves nine”, so goes the adage. Applied to classroom management, this means that we _______. A. must be reactive in our approach to discipline B. may not occupy ourselves with disruptions which are worth ignoring because they are minor C. have to resolve minor disruptions before they are out of control D. may apply 9 rules out of 10 consistently 48. In Krathwoh’s taxonomy of objectives in the affective, which is most authentic? A. Responding C. Valuing B. Organization D. Characterization 49. Teacher G’s lessons objective has something to do with the skill of synthesizing? Which behavioral term is most appropriate? A. Test B. Appraisal C. Assess D. Theorize 50. Teacher H strives to draw participation of every student into classroom discussion. Which student’s need is she trying to address? The need A. to get everything out in the open B. to feel significant and be part of a group C. to be creative D. to show their oral abilities to the rest of the class 51. A sixth grade twelve-year old boy comes from a dysfunctional family and has been abused and neglected. He has been to two orphanages and three different elementary schools. The student can decode on the second grade level, but he can comprehend orally material at the fourth or fifth grade level. The most probable cause/s of this student’s reading problem is/are ______. A. poor teaching C. neurological factors B. emotional factors D. immaturity 52. Teacher B clears his throat to communicate disapproval of a student’s behavior. Which specific influence technique is this? A. Proximity control C. Signal interference B. Direct appeal D. Interest boosting 53. Which questioning practice promotes more class interaction? A. Focusing on convergent questions. B. Focusing on divergent questions. C. Asking rhetorical questions D. Asking the question before calling a student 54. The following are used in writing performance objectives EXCEPT A. integrate B. diagram C. delineate D. comprehend 55. Research tells that teachers ask mostly content questions. Which of the following terms does NOT refer to content question? A. Direct B. Convergent C. Concept D. Closed 56. Which is behavioral term describes a lesson outcome in the highest level of Bloom’s cognitive domain? A. Design B. Create C. Evaluate D. Analyze 57. Which is an appropriate way to make manage off-task behavior? A. Make eye contact. B. Stop your class activity to correct a child who is no longer on task. C. Move closer to the child. D. Redirect a child’s attention to task and check his progress to make sure he is continuing to work. 58. How can you exhibit referent power on the first day of school? A. By reminding your students your authority over them again and again. B. By giving your students a sense of belonging and acceptance. C. By making them feel you know what you are talking about. D. By telling them the importance of good grades. 59. If a teacher plans a constructivist lesson, what will he most likely do? Plan how he can _______. A. do evaluate his student’s work B. do reciprocal teaching C. do lecture to his students D. engage his students in convergent thinking 60. How can you exhibit expert power on the first day of school? A. By making them realize the importance of good grades. B. By giving your students a sense of belonging and acceptance. C. By reminding then your students your authority over them again and again. D. By making them feel you know what you are talking about. 61. Read this question: “How will you present the layers of the earth to your class?” This is a question that _________. A. assesses cognition C. probes creative thinking B. directs D. leads the students to evaluate 62. In Krathwoh’s affective domain of objectives, which of the following is the lowest level of effective behavior? A. Characterization C. Organization B. Responding D. Valuing 3
  • 4. 63. The following are sound specific purpose of questions EXCEPT: A. To stimulate learners to ask questions B. To arouse interest and curiosity C. To call the attention of an inattentive student D. To teach via student answers 64. An effective classroom manager uses low-profile classroom control. What is a low-profile classroom technique? A. Note to parents C. Withdrawal of privileges B. Raising the pitch of the voice D. After-school detention 65. Test norms are established in order to have a basis for ______. A. identifying pupil’s difficulties C. establishing learning goals B. computing grades D. interpreting test results 66. Quiz to formative test while periodic is to _________. A. criterion-reference test C. norm-reference test B. summative test D. diagnostic test 67. Two students are given the WISC III. One has a full scale IQ of 91, while the other has an IQ of 109. Which conclusion can be drawn? A. The first student is probably below average, while the second has above average potential. B. Both students are functioning in the average range of intellectual ability C. Another IQ test should be given to truly assess their intellectual potential. D. The second student has significantly higher intellectual ability. 68. Student’s scores on a test were: 72,72,73,74,76,78,81,83,85. The score 76 is the ___. A. mode B. mean C. median D. average 69. The best way for a guidance counselor to begin to develop study skills and habits in underachieving student would be to ______. A. have them view filmstrips about various study approaches B. encourage students to talk study habits from their own experiences C. give out list of effective study approaches D. have these underachieving students observe the study habits of excelling students. 70. Which describes norm-referenced grading? A. What constitutes a perfect score C. An absolute standard B. The performance of the group D. The student’s past performance 71. Standard deviation is to variability as mode to _____. A. correction C. central tendency B. discrimination D. level of difficulty 72. If teacher wants to test student’s ability to organize ideas, which type of test should she formulate? A. Short answer C. Essay B. Technical problem type D. Multiple-choice type 73. What is the mean of this score distribution 4,5,6,7,8,9,10? A. 6 B. 7 C. 8.5 D. 7.5 74. Study this group of tests which was administered with the following results, then answer the question. Subject Mean SD Ronnel’s scrore Math 56 10 43 Physics 41 9 31 English 80 16 109 In which subject(s) did Ronnel perform best in relation to the group’s performance? A. Physics B. English C. Math D. Physics and Math 75. Out of 3 distracters in a multiple choice test items, namely B,C, and D, no pupil choice D as answer. This implies that D is ________. A. a plausible distracter B. a vague distracter C. an effective distracter D. an ineffective distracte 76. Shown a picture of children in sweaters inside the classrooms, the students were asked this question: In what kind of climate do these children live? This is a thought questions on ______. A. predicting B. applying C. creating D. inferring 77.Which one can best evaluate student’s attitudinal development? A. Short answer test C. Essay test B. Observation D. Portfolio 78. In which competency do my students find the greater difficulty? In item with a difficulty index of A. 0.9 B. 0.1 C. 1.0 D. 0.5 79. With synthesizing skills in mind, which has the highest diagnostic value? A. Performance test C. Completion test B. Multiple choice test D. Essay test 80. “In the light of the facts presented, what is most likely to happen when…?” is a simple thought question on _____. A. synthesizing B. generalizing C. inferring D. justifying 81. Which is most implied by negatively skewed score distribution? A. The score are evemly distributed from left to the right. B. Most pupils are achievers. C. Most of the scores are low. D. Most of the scores are high. 82. Study this group of tests which was administered with the following results, then answer the question. Subject Mean SD Ronnel’s scrore Math 56 10 43 Physics 41 9 31 English 80 16 109 In which subject(s) were the scores most homogenous? A. English C. Physics and Math B. Math D. Physics 83. Which guidance in test construction is NOT observed in this test item “Jose Rizal wrote ____”. A. The alternative must be plausible. B. There must be only one correct answer. C. The central problem should be packed in the stem. D. Alternatives must grammatical parallelism. 84. Are percentile ranks the same as percentage correct? A. Yes B. It cannot be determined unless scores are given. C. It cannot be determined unless the number of examinees is given D. No 85. “What is most likely to happen to our economy when export continuously surpasses import” is a thought question on ______. A. creating C. synthesizing B. relating cause-and-effect D. predicting 4
  • 5. 86. The burnout malady gets worse if a teacher doesn’t intervene to change whatever areas he or she in control. Which one can renew a teacher’s enthusiasm? A. Judge someone else as wrong. C. Stick to job.B. Initiate changes in jobs. D. Engage in self-pity. 87. In the context on the theory on multiple intelligences, what is one weakness of the paper-pencil test? A. It lacks reliability. B. It is not easy to administer. C. It utilizes so much time. D. It puts the non-linguistically intelligent at a disadvantage 88. Which holds true to standardized tests? A. They are scored according to different standards. B. They are administered differently. C. They are used for assigned grades. D. They are used for comparative purposes. 89. The first thing to do in constructing a periodic test is for a teacher to A. decide on the type of test to construct B. study the content C. decide on the number of items for the test D. go back to her instructional objectives 90. Which can effectively measure student’s awareness of values? A. Moral dilemma C. Anecdotal record B. Projective techniques D. Likert scales 91. I drew learners into several content areas and encouraged them to solve a complex question for interdisciplinary teaching. Which strategy did I use? A. Thematic instruction C. Unit method B. Problem-centered learning D. Reading-writing activity 92. With specific details in mind, which one has (have) a stronger diagnostic value? A. Restricted essay test B. Non-restricted essay test C. Restricted and non-restricted essay tests D. Multiple choice test 93. If your Licensure Examination for Teacher (LET) items sample adequately the competencies listed in the syllabi, it can be said that LET possesses ______ validity. A. concurrent B. content C. predictive D. construct 94. In a criterion-referenced testing, what must you do to ensure that your test is fair? . A. Ask each student to contribute one question. B. Use objectives for the unit as guide in your test construction. C. Make twenty questions but ask the students to answer only ten of their choice. D. Make all of the questions true or false. 95. In his second item analysis, Teacher H found out that more from the lower group got the test item # 6 correctly. This means that the test item _______ A. has a lower validity. B. has a positive discriminating power. C. has a high reability D. has a negative discriminating power 96. Study this group of tests which was administered with the following results, then answer the question. Subject Mean SD Ronnel’s scrore Math 56 10 43 Physics 41 9 31 English 80 16 109 In which subject(s) did Ronnel perform poorly in relation to the group’s performance? A. Physics C. English B. Math D. English and Math 97. To promote effective practice, which guidelines should you bear in mind? Practice should be ______. A. done in an evaluative atmosphere B. difficult for students to learn a lesson C. arranged to allow student to receive feedback D. take place over a long period of time 98. Teacher A discovered that his pupils are very good in dramatizing. Which tool must have helped him discover his pupils’ strength? A. Portfolio assessment C. Performance test B. Paper-and-pencil test D.Journal entry 99. By what name is Socratic method also known? A. Mastery learning C. Questioning Method B. Morrison method D. Indirect instruction 100. Teacher F wanted to teach the pupils the skills to do cross stitching. He check up quiz was a written test on the steps of cross stitching. What characteristic of a good test does it lack? A. Reliability B. Scorability C. Objectivity D. Validity 101. Which method has been proven to be effective in courses that stress acquisition of knowledge? A. Cooperative learning C. Mastery learning B. Socratic method D. Indirect instruction 102. Teacher Y does norm-referenced interpretation of scores. Which of the following does she do? A. She describes group performance in relation to a level of mastery set. B. She uses a specified content as its frame of reference. C. She compares every individual student’s scores with others’ scores. D. She describes what should be their performance. 103 . Which is a major advantage of a curriculum-based assessment? A. It is based on an norm-referenced measurement model. B. It tends to focus on anecdotal information on student progress. C. It connects testing with teaching. D. It is informal in nature. 104. Which is the first step in planning an achievement test? A. Select the type of test items to use. B. Decide on the length of the test C. Build a table of specification D. Define the instructional objective 105. Which test has broad sampling of topics as strength? A. Short answer test C. Problem test B. Objective test D. Essay test 106. Which is one characteristic of an effective classroom management? A. It respects cultural norms of a limited group students. B. It quickly and unobtrusively redirects misbehavior once it occurs. C. Strategies are simple enough to be used consistently. D. It teaches dependence on other for self-control. 107. Which guideline must be observed in the use of prompting to shape the correct performance of your students? A. Refrain form using prompts. B. Use the most intrusive prompt first. C. Use the least intrusive prompt first. 5
  • 6. D. Use all prompts available. 108. To elicit more student’s response, Teacher G. made use of covert responses. Which one did she NOT do? A. She refrained from judging on the student’s responses. B. She had the students write their responses privately then called each of them. C. She showed the correct answers on the overhead after the students have written their responses. D. She had the students write their response privately. 109. In self-directed learning, to what extent should a teacher’s “scaffolding” be? A. None, to force the student to learn by himself B. To the maximum, in order to extend to the student all the help he needs. C. To the minimum, to spend up development of student’s sense of independence. D. To a degree the student needs it. 110. Read the following then answer the questions: TEACHER: IN WHAT WAYS OTHER THAN THE PERIODIC TABLE MIGHT WE PREDICT THE UNDISCOVERED ELEMENTS? BOBBY: WE COULD GO TO THE MOON AND SEE IF THERE ARE SOME ELEMENTS THERE WE DON’T HAVE. BETTY: WE COULD DIG DOWN TO THE CENTER OF THE EARTH AND SEE IF WE FIND ANY OF THE MISSING ELEMENTS. RICKY: WE COULD STUDY DEBRIS FROM THE METEORITES- IF WE CAN FIND ANY. TEACHER: THOSE ARE ALL GOOD ANSWERS BUT WHAT IF THOSE EXCURSIONS TO THE MOON, TO THE CENTER OF THE EARTH OR TO FIND METEORITES WERE TOO COSTLY AND TIME CONSUMING? HOW MIGHT WE USE THE ELEMENTS WE ALREADY HAVE HERE ON EARTH TO FIND SOME NEW ONES? Question: which questioning strategy/ies does/do the exchange of thoughts above illustrate? A. Extending and lifting B. Funneling C. Sowing and reaping D. Nose-dive 111. Which are direct measures of competence? A. Standardized tests B. personality tests C. Performance tests D. paper-and-pencil tests 112. Availment of the Philippine Education Placement Test for adults and out-of-school youths is in support of the government’s educational program towards ___ . A. relevance B. quality and relevance C. equitable access D. quality 113. Teacher T taught a lesson denoting ownership by means of possessives. He first introduced the rule, then gave examples, followed by class exercises, then back to the rule before he moved into second rule. Which presenting technique did he use? A. Combinatorial B. Sequential C. Comparative D.. Part-whole 114. Which Filipino trait works against the shift in teacher’s role from teacher as a fountain of information to teacher as facilitator? A. Pakikisama B. Authoritarianism C. Authoritativeness D. Hiya 115. Referring to Teacher S, Nicolle describes her teacher as “fair, caring and someone you can talk to”. Which power or leadership does Teacher S have? A. Expert power B. Reward power C. Referent power D. Legitimate power 116. Teacher B uses the direct instruction strategy. Which sequence of steps will she follow? I. Independent practice II. Feedback and correctiveness III. Guided student practice IV. Presenting and structuring V. Reviewing the previous day’s work A. III-II-IV-I-V B. I-V-II-III-IV C. V-II-IV-III-I D. V-IV-III-II-I 117. The teacher’s first task in the selection of media in teaching is to determine the A. technique to be used B. availability of the media C. objectives of the lesson D. choice of the students 118. Teacher M’s pupils are quite weak academically and his lesson is already far behind time table. How should Teacher M proceed with his lesson? A. Experientially B. Deductively C. Inductively D. Logically 119In mastery learning, the definition of an acceptable standard of performance is called a A. behavior B. SMART C. condition D. criterion measure 120. What values are being given priority by juries in criminal cases? A. The rights of the criminal over the strict interpretation of the law B. The safety of the community over sympathy for the criminal C. The punishment of the criminal over the safety of the community D. The needs of the criminals over the advice of the judge 121. Helping in the development of graduates who are “maka-Dios” is an influence of ___. A. dialectical morality C. situational morality B. naturalistic morality D. classical Christian morality 122. What should you do if a parent who is concerned about a grade his child received compared to another student’s grade, demands to see both students’ grades? A. Refuse to show any record without expressing permission from principal. B. Show only his child’s record. C. Show both records to him. D. Refuse to show either record. 123. A teacher / student is held responsible for his actions because s/he _______. A. has reason B. has instincts C. has a choice D. is mature 124. With indirect instruction in mind, which does NOT belong to the group? A. Problem solving C. Lecture-recitation B. Discovery D. Inductive reasoning 125. A teacher who equates authority with power does NOT usually __________. A. retaliate C. shame B develop self-respect in every pupil D. intimidate 126. The typical autocratic teacher consistently does the following EXCEPT A. ridiculing students C. encouraging students B. intimidating students D. shaming students 127. Teacher H and teacher I are rivals for promotion. To again the 6
  • 7. favor of the promotion staff, teacher I offers her beach resort for free for members of he promotional staff before ranking. As one of the contenders for promotions, is this becoming of her to do? A. Yes. The rare invitation will certainly be welcomed by an overworked promotional staff B. Yes. There’s nothing wrong with sharing one’s blessings. C. Yes. This will be professional growth for the promotional staff D. No. This may exert undue influence on the members of the promotional staff and so may fail to promote on the basis of merit. 128. Teacher A knows of the illegal activities of a neighbor but keeps quiet in order not to be involved in any investigation. Which foundational principle of morality does Teacher A fail to apply? A. Between two evils, do the lesser evil B. The principle of double effect C. The end does not justify the means D. Always do what is right 129. Which is a true foundation of the social order? A. Equitable distribution of wealth B. Strong political leadership C. The reciprocation of rights and duties D. Obedient citizenry 130. Teacher F is a newly converted to a religion. Deeply convinced of his new found religion, he starts Monday classes by attacking one religion and convinces his pupil to attend their religion services on Sundays. Is this in accordance with the Code of Ethics of Professional Teachers? A. Yes. In the name of academic freedom, a teacher can decide what to teach. B. Yes. What he does strengthens value education. C. No. A teacher should not use his position to proselyte others. D. Yes. What he does is a value education. 131. Teacher Q does not want Teacher B to be promoted and so writes an anonymous letter against Teacher B accusing her of fabricated lies. Teacher Q mails this anonymous letter to School Division Superintendent. What should Teacher Q do if she has to act professionally? A. Hire a group to distribute poison letters against Teacher B for the information dissemination. B. Submit a signed justifiable criticism against Teacher B, if there is any. C. Go straight to the School Division Superintendent and gives criticism verbally. D. Instigate student activists to read poison letter over the microphone 132. Rights and duties are correlative. This means that. A. rights and duties arise from natural law B. rights and duties regulate the relationship of men in society C. rights and duties ultimately come from God D, each right carries with it one or several corresponding duties 133. In the Preamble of Code of Ethics of Professional Teachers, which is NOT said of teachers? A. Possess dignity and reputation B. With high moral values as well as technical and professional competence C. Duly licensed professionals D. LET passer 134. In what way can teachers uphold the highest possible standard of quality education? A. By working out undeserved promotions B. By wearing expensive clothes to change people’s poor perception of teachers C. By putting down other professions to lift the status of teaching D. By continually improving themselves personally and professionally 135. If you agree with Rizal on how you can contribute to our nation’s redemption, which should you work for? A. Gaining economic recovery B. Stabilizing the political situation C. Opening our doors to foreign influence D. Upgrading the quality of the Filipino through education 136. In a study conducted, the pupils were asked which nationality they preferred. If given a choice. Majority of the pupils wanted to Americans. In this case, in which obligation, relative to the state, do schools seem to be failing? In their obligation to A. respect for all duly constituted authorities B. install allegiance to the Constitution C. promote obedience to the laws of the state D. promote national pride 137. History books used in schools are replete with events portraying defeats and weaknesses of the Filipino as a people. How should you tackle them in the classroom? A. Present them and express your feelings of shame. B. Present facts and use them as means in inspiring your class to learn from them. C. Present them and blame those people responsible or those who have contributed. D. Present them as they are present and tell the class to accept reality. 138. Teacher often complain of numerous non-teaching assignments that adversely affect their teaching. Does this mean that teachers must be preoccupied only with teaching? A. Yes, because other community leaders, not leaders, not teachers, are asked to lead in community activities. B. Yes, because teaching is enough full time job. C. Yes, if they are given other assignments, justice demands that they be properly compensated D. No, because every teacher is expected to provide leadership and initiative in activities for betterment of communities. 139. Each teacher is said to be a trustee of the cultural and educational heritage of the nation and is under obligation to transmit to learners such heritage. Which practice makes him fulfill such obligation? A. Observing continuing professional education. B. Use the latest instruction technology. C. Study the life of Filipino heroes. D. Use interactive teaching strategies 140. Teacher A is directed to pass an undeserving student with a death threat. Which advice will a hedonist give? A. Pass the student. That will be use to the student, his parents and you. B. Don’t pass him. You surely will not like someone to give you a death threat in order to pass. C. Pass the student. Why suffer the threat? D. Don’t pass him. Live by principle of justice. You will get reward, if not in this life, in the next. 141. Teacher P wants to develop the skill of synthesizing in her pupil. Which one will she do? A. Directs her students to ask questions on the parts of the lesson not understood. B. Tell her pupils to state data presented in graphs. C. Ask her students to formulate a generalization from the data shown in graphs. D. Ask her students to answer questions beginning with “What if …” 142. Which measure(s) of central tendency separate(s) the top half of the group from the bottom half? A. Median and mean B. Mode C. Mean D. Media 143. Which applies when skewness is zero? 7
  • 8. A. Median is greater than mean. B. Mean is greater than the median. C. Scores have three modes. D. Scores are normally distributed. 144. Based on Edgar Dale’s Cone of Experience, which activity is farthest from the real thing? A. Attend exhibit B. View images C. Hear D. Read 145. NSAT and NEAT results are interpreted against set mastery level. This means that NSAT and NEAT fall under _______. A. aptitude test C. criterion-referenced test B. norm-referenced test D. intelligence test 146. Which one can enhance the comparability of grades? A. Individual teachers giving weights to factors considered for rating. B. Formulating tests that vary from one teacher to another. C. Allowing individual teachers to determine factors for rating D. Using common conversation table for translating test scores in to ratings. 147. Read the following then answer the questions: TEACHER: IN WHAT WAYS OTHER THAN THE PERIODIC TABLE MIGHT WE PREDICT THE UNDISCOVERED ELEMENTS? BOBBY: WE COULD GO TO THE MOON AND SEE IF THERE ARE SOME ELEMENTS THERE WE DON’T HAVE. BETTY: WE COULD DIG DOWN TO THE CENTER OF THE EARTH AND SEE IF WE FIND ANY OF THE MISSING ELEMENTS. RICKY: WE COULD STUDY DEBRIS FROM THE METEORITES-IF WE CAN FIND ANY. TEACHER: THOSE ARE ALL GOOD ANSWERS. BUT WHAT IF THOSE EXCURSIONS TO THE MOON, TO THE CENTER OF THE EARTH OR TO FIND METEORITES WERE TOO COSTLY AND TIME CONSUMING? HOW MIGHT WE USE THE ELEMENTS WE ALREADY HAVE HERE ON EARTH TO FIND SOME NEW ONES? Question: The Teacher questions in the above exchange are examples of ____ questions. A.Direct B. Concept C. Closed D. Fact 148. I combined several subject areas in order to focus on a single concept for interdisciplinary teaching. Which strategy/method did I use? A. Reading–writing activity C. Thematic instruction B. Unit method D. Problem-entered learning 149. “All men are pretty much alike. It is only by custom that they are set apart, “ said one Oriental philosopher. Where can this thought be most inspiring? A. In a class composed of indigenous people. B. In heterogeneous class of learners. C. In a multi-cultural group of learners. D. In a multi-cultural and heterogeneous groups of learners and indigenous peoples’ group. 150. I want to teach concepts, patterns and abstractions. Which method is most appropriate? A. Discovery C. Direct instruction B. Problem solving D. Indirect instruction 151. What should a teacher do for students in his class who are on grade level? A. Mohammed B. Confucius C. Buddha D. Lao tsu 152. Who among the following needs less verbal counseling but needs more concrete operational forms of assistance? The child who ______. A. has attention-deficit disorder C. has mental retardation B. has learning disability D. has conduct disorder 153. Teacher B is a teacher of English as Second Language: she uses vocabulary cards, fill-in-the blank sentences, dictation and writing exercises in teaching a lesson about grocery shopping. Based on this information, which of the following is a valid conclusion? A. The teacher is emphasizing listening and speaking skills. B. The teacher wants to do less talk. C. The teacher is reinforcing learning by giving the same information in a variety of methods. D. The teacher is applying Bloom’s hierarchy of cognitive learning. 154. Which illustrates a development approach in guidance and counseling? A. Acting as a mentor B. Making the decision for the confused student C. Spotting on students in need of guidance D. Teaching students how to interact in a positive manner 155. Which technique should a teacher use to encourage response if his students do not respond to his question? A. Ask a specific student to respond, state the question, and wait a response. B. Tell the class that it will have detention unless answers are forthcoming. C. Wait for a response. D. Ask another question, an easier one. 156. In the parlance of test construction what does TOS mean? A. Table of Specifics C. Table of Specification B. Term of Specifications D. Table of Specific Test Items 157. Based on Edgar Dale’s Cone of Experience, which activity is closest from the real thing? A. View images B. Hear C. Watch a demo D. Attend exhibit 158. For which may you use the direct instruction method? A. Become aware of the pollutants around us. B. Use a microscope properly. C. Distinguish war from aggression. D. Appreciate Milton’s Paradise Lost. 159. Why should a teacher NOT use direct instruction all the time? A. It requires much time. B. It is generally effective only in the teaching of concepts and abstractions. C. It requires use of many supplemental materials. D. It reduces student’s engagement in learning. 160. What measure/s of central tendency do/es the number 16 represent in the following data: 14,15,17,16,19,20,16,14,16? A. Median B. Mode and mean C. Mode D. Median 161. Teacher E discussed how electricity flows through wires and what generates the electric charge. Then she gave the students wires, bulbs, switches, and dry cells and told the class to create a circuit that will increase the brightness of each bulb. Which one best describes the approach used? A. It helped students understand scientific methodology. B. It was contructivist. C. It used a taxonomy of basic thinking skills. D. It used cooperative learning. 162. Teacher B is a teacher of English as Second Language: she uses vocabulary cards, fill-in-the blank sentences, dictation and writing 8
  • 9. exercises in teaching a lesson about grocery shopping. Based on this information, which of the following is a valid conclusion? A. The teacher wants to make her teaching easier by having less talk. B. The teacher is emphasizing reading and writing skills. C. The teacher is applying Bloom’s hierarchy of cognitive learning D. The teacher is teaching in a variety of ways because not all students learn in the same manner. 163. Standard deviation is to variability as mean is to _______. A. level of difficulty C. coefficient of correlation B. central tendency D. discrimination index 164. Which is a form of direct instruction? A. Problem solving C. Programmed instruction B. Discovery process D. Inductive reasoning 165. Value clarification as a strategy in Values Education classes is anchored on which philosophy? A. Hedonism C. Idealism B. Existentialism D. Christian philosophy 166. Which type of reports to “on-the spot” description of some ncident, episode or occurrence that is being observed and recorded as being of possible significance? A. Value and interest report C. Biographical report B. Anecdotal report D. Autobiographical report 167. Direct instruction id a facts, rules, and actions as indirect instruction is for _____ ,____, _____. A. hypotheses, verified data and conclusions B. concepts, processes and generalizations C. concepts, patterns and abstractions D. guesses, data and conclusions 168. Which does NOT belong to the group of alternative learning systems? A. Multi-grade grouping C. Multi-age grouping B. Non-graded grouping D. Graded education 169. Which activity should a teacher have more for his students if he wants to develop logical-mathematical thinking? A. Choral reading C. Problem solving B. Storytelling D. Drama 170. Which behavior is exhibited by a student who is strong in interpersonal intelligence? A. Works on his/her own. B. Keeps interest to himself/herself C. Seeks out a classmate for help when problem occurs. D. Spends time meditating. 171. If teacher has to ask more higher-order questions, he has to ask more ____ questions. A. closed B. Fact C. convergent D. concept 172. Which criterion should guide a teacher in the choice of instructional devices? A. Novelty B. Appropriateness C. Cost D. Attractiveness 173. The primary objective of my lesson is: “ To add similar fractions correctly.” This is the advice of the ______. A. Primary B. Major C. Terminal D. Enabling 174. With-it-ness, according to Kourin, is one of the characteristics of an effective classroom manager. Which phrase goes with it? A. Have hands that write fast. B. Have a mouth ready to spea C. Have eyes on the back of your hands D. Have minds packed with knowledge. 175. Which is one role play in the pre-school and early childhood years? A. Separates reality from fantasy. B. Develops competitive spirit. C. Develops the upper and lower limbs D. Increase imagination due to expanding knowledge and emotional range. 176. A guest in one graduation rites told his audience: “Reminder, you are what you choose to be”. The guest speaker is more of a/an _____. A. idealism B. existentialist C. realistic D. pragmatist 177. Principal C shares this thought with the teachers: Subject matter should help students understand and appreciate themselves as unique individual who accept complete responsibility for their thoughts, feelings and actions. From which philosophy is this thought based? A. Essentialism B. Perennialism C. Progressivism D. Existentialism 178. Which of the following prepositions is attributed to Plato? A. Sense perception is the most accurate guide to knowledge. B. Learning is the discovery of truth as latent ideas are brought to consciousness C. Truth is relative to a particular time and place. D. Human beings create their own truths. 179. Student B claims: “I cannot see perfection but I long for it. So it must be real. “under which group can he be classified? A. Realist B. Empiricist C. Pragmatist D. Idealist 180. Teacher U teaches to his pupils that pleasure is not the highest good. Teacher’s teaching is against what philosophy? A. Realism B. Hedonism C. Empiricism D. Epicureanism 181. Who among the following puts more emphasis on core requirements, longer school day, longer academic year and more challenging textbooks? A. Progressivist B. Perennialist C. Existentialist D. Essentialist 182. To come closer to the truth we need to “go back to the things themselves.” This is the advice of the ______. A. pragmatists B. idealist C. phenomenologists D. behaviorist 183. Teacher W wants to review and check on the lesson of the previous day. Which one will be most reliable? A. Sampling the understanding of a few students B. Explicitly reviewing the task-relevant information necessary for the day’s lesson. C. Having students correct each other’s work. D. Having students identify difficult homework problems. 184. Whose influence is the education program that puts emphasis on self-development through the classics, music, and ritual? A. Mohammed B. Confucius C. Buddha D. Lao tsu 185. Teacher H gave first-grade class a page with a story in which picture take the place of some words. Which method did she use? A. The Spaulding method B. The language experience approach C. The rebus method D. The whole language approach 9
  • 10. 186. The search for related literature by accessing several data bases by the use of a telephone line to connect a computer that have database is termed ______. A. computer search C. manual search B. on-line search D. compact disc search 187. On whose philosophy was A. S. Neil Summerhill, one of the most experimental schools based? A. Pestalozzi B. John Locke C. Montessori D. Rousseau 188. You arrive at knowledge by re-thinking of latent ideas. From whom does this thought come? A.Existentialism B. Experimentalist C. Idealist D. Realist 189. As a teacher, you are rationalist, Which among these will be your guiding principle? A. I must teach the child that we can never have real knowledge of anything. B. I must teach the child to develop his mental powers to the full. C. I must teach the child so he is assured of heaven D. I must teach the child every knowledge, skills and value that he needs for a better future. 190. Principal B tells her teachers that training in the humanities is most important. To which educational philosophy does he adhere? A. Progressivism B. Existentialism C. Essentialism D. Perennialism 191. As a teacher, you are reconstructionist, Which among these will be your guiding principle? A. I must teach the child so he is assured of heaven B. I must teach the child every knowledge, skills and value that he needs for a better future. C. I must teach the child to develop his mental powers to the full. D. I must teach the child that we can never have real knowledge of anything. 192. What can be said of Peter who obtained a score of P75 in a Grammar objective test? A. His rating is 75. B. He answered 75 items in the test correctly. C. He answered 75% of the test items correctly. D. He performed better that 5% of his classmate. 193. Which group of philosophers maintain the “truth exists in an objective order that is independent of the knower”? A. Idealists B. Pragmatists C. Existentialists D. Realists 194. If we teach our students to think creativity, what do we encourage them to do? A. To question the illogical B. To criticize the unreasonable C. Think “within the box” D. Do “outside-the-box” thinking 195. Which of the following is considered a peripheral device? A. Keyboard B. CPU C. Monitor D. Printer 196. Helping in the development of graduates who are “maka-Diyos” is an influence of ___. A. naturalistic morality B. situational morality C. dialectical morality D. classical Christian morality 197. In research which is another term for independent variable? A. Response B. Outcome C. Criterion D. Input 198. Which practices speak of education of the human spirit? I. Familiarizing the learner with the world’s heritage of art in all its forms II. Encouraging the learner to be practical III. Studying the biographies of heroes and martyr IV. Teaching the learner how to learn A. I, II B. I,IV C. II,III D. I, III 199. Rights cannot be unduly tramped upon or suppressed without moral guit because they are A. Inalienable. C. perfect. B. Inviolable. D. adventitious. 200. Which instructional material is closest to direct experience? A. Film showing C. Field trip B. Simulation D. Dramatization 10
  • 11. 11